You are on page 1of 48

Limits 11.

CHAPTER

11 Limits

Chapter Highlights
Limit of a function, Indeterminate forms, Algebra of limits, Evaluation of limits, Algebraic limits, Limit of an
algebraic function when x → ∞, Trigonometric limits, Exponential and logarithmic limits, Evaluation of limits
using L’Hospital’s rule

LIMIT OF A FUNCTION Method for Finding Right Hand Limit


Let a function f be defined at every point in the To evaluate lim f ( x )
x → a+
neighbourhood of a (an open interval about a) except pos-
sibly at a. If as x approaches closer and closer to a, but not 1. Put x = a + h in f (x) to get
equal to a, then the value of the function f (x) approaches a 2. Take the limit as h → 0.
real number l. The number l is referred to as the limit of f (x)
as x tends to a and we write it as Left Hand Limit
lim f ( x ) = l We say that left hand limit of f (x) as x tends to ‘a’ exists
x→a and is equal to l2 if as x approaches ‘a’ through values less
Note that f (x) approaches l means the absolute difference than ‘a’, the values of f (x) approach a definite unique real
between f (x) and l, i.e. |f (x) – l| can be made as small as number l2 and we write
we please.
When the values of f (x) do not approach a single lim f ( x ) = l2
x→ a-
finite value as x approaches a, we say that the limit Does
not exist. or f (a – 0) = l2

CAUTION Method for Finding Left Hand Limit


To evaluate lim f ( x ).
A number is said to be a limiting value only if it is finite and x→ a-
real, otherwise we say that the limit does not exist. 1. Put x = a – h in f (x) to get lim f ( a + h).
h→ 0
2. Take the limit as h → 0.
Right Hand Limit
We say that right hand limit of f (x) as x tends to ‘a’ exists CAUTION
and is equal to l1 if as x approaches ‘a’ through values
greater than ‘a’, the values of f (x) approach a definite For finding lim f( x ), we study the behaviour of the function f
x →a
unique real number l1 and we write in the neighbourhood of ‘a’ and not at ‘a’. Thus, the function
f may or may not be defined at x = a.
lim f ( x ) = l1 or f (a + 0) = l1
x → a+
11.2 Chapter 11

and x – ai > 0 for i = 1, 2, …, m – 1


SOLVED EXAMPLES
x - ai
x 2 - 9 x + 20 \ Ai = = –1, for i = m, m + 1, …, n
1. lim = - ( x - ai )
x→5 x - [ x]
x - ai
(A) 1 and Ai = = 1, for i = 1, 2, …, m – 1
x - ai
(B) 0
(C) Does not exist Similarly, if x is in the right neighbourhood of ai
(D) Cannot be determined Then x – ai < 0 for i = m + 1, …., n and x – ai > 0
for i = 1, 2, …, m
Solution: (C) x - ai
\ Ai = = – 1 for i = m + 1, … n
x 2 - 9 x + 20 ( x - 4)( x - 5) - ( x - ai )
lim = lim
x→5 x - [ x] x→5 x - [ x] x - ai
and Ai = = 1 for i = 1, 2, …, m
( x - 4)( x - 5) x - ai
LHL = lim
x → 5- x - [ x] Now, lim ( A1 A2 ... An ) = (–1)n – m + 1
x → am-
(5 - h - 4)(5 - h - 5)
= lim (h > 0)
h→0 (5 - h) - [5 - h] and lim ( A1 A2 ... An ) = (–1)n – m
x → am+
(1 - h)( - h) (1 - h)( - h)
= lim = lim Hence, lim ( A1 A2 ... An ) Does not exist.
h→0 5-h-4 h→0 (1 - h) x → am

\ LHL = 0 INDETERMINATE FORMS


( x - 4)( x - 5)
Also, RHL = lim If a unique value cannot be assigned to f (a), then f (x) is
x→5 x - [ x]
said to be inderminate at x = a.
(5 + h - 4)(5 + h - 5) 0
= lim Most general of all indeterminate forms is , others
h→0 (5 + h) - [5 + h] being 0
(1 + h)( h) (1 + h)h 1 1 0-0 0
= lim = lim 1. ∞ – ∞ = - = = which is indeterminate
h→0 5+h-5 h→0 h 0 0 0 0
and hence is (∞ – ∞)
\ RHL = 1
∞ 1/ 0 0
As LHL ≠ RHL 2. = = which is indeterminate and hence is
∞ 1/ 0 0
\ Limit does not exist. ⎛ ∞⎞
x - ai ⎜⎝ ⎟⎠

2. If Ai = , i = 1, 2, …, n and if a1 < a2 < a3 < … < an.
| x - ai | 1 0
3. 0 × ∞ = 0. = which is indeterminate and hence is
Then lim ( A1 A2 ... An ) , 1 ≤ m ≤ n (0 × ∞)
0 0
x → am

(A) is equal to (–1)m 4. 1∞


(B) is equal to (–1)m + 1 Let y = 1∞
(C) is equal to (–1)m – 1 ⇒ log y = ∞ log 1 = ∞ × 0 which is indeterminate
(D) Does not exist and hence is 1∞

Solution: (D) 5. 00
x - ai Let y = 00
We have, Ai = , i = 1, 2, …, n ⇒ log y = 0 ⋅ log 0 = 0 × ∞ which is indeterminate
| x - ai |
and hence is 0º
and a1 < a2 < … an – 1 < an. 6. ∞º
Let x be in the left neighbourhood of am. Let y = ∞º
Then, ⇒ log y = 0 ⋅ log ∞ = 0 × ∞ which is indeterminate
x – ai < 0 for i = m, m + 1, … n and hence is ∞º.
Limits 11.3

ALGEBRA OF LIMITS Solution: (B)


Since, 0 ≤ (rx) < 1 for r = 1, 2, 3, …, n
If lim f ( x ) = l and lim g ( x ) = m, then following results n n
x→a x→a
are true: ⇒ 0≤ ∑ (rx) < ∑ (1)
r =1 r =1
1. lim [ f ( x ) + g ( x )] = lim f ( x ) + lim g ( x ) = l + m. n
x→a x→a x→a ⇒ 0≤ ∑ (rx) < n
2. lim [ f ( x ) - g ( x )] = lim f ( x ) – lim g ( x ) = l – m. r =1
x→a x→a x→a
Dividing throughout by n2, we have
3. lim k ⋅ f ( x ) = k ⋅ lim f ( x ) = kl, n
x→a

where k is a constant.
x→a
∑ (rx)
0 r =1 1
≤ <
4. lim [ f ( x ) ⋅ g ( x )] = lim f ( x ) ⋅ lim g ( x ) = lm. n 2
n 2 n
x→a x→a x→a n
lim f ( x )
⎡ f ( x) ⎤ x → a l ∑ (rx)
5. lim ⎢ ⎥ = = (provided m ≠ 0). ⇒ lim 0 ≤ lim
r =1
< lim
1
x → a ⎣ g ( x) ⎦ lim g ( x ) m
x→a n→∞ n→∞ n2 n→∞ n
⎛ ⎞ n
6. lim ( fog ) ( x ) = lim f [ g ( x )] = f ⎜ lim g ( x )⎟ =
x→a x→a ⎝ x→a ⎠ ∑ (rx)
f (m). r =1
⇒ 0 ≤ lim <0
n→∞ n2
In particular,
( x ) + (2 x ) + ... + ( nx )
⎛ ⎞ \ 0 ≤ lim <0
(a) lim log g ( x ) = log ⎜ lim g ( x )⎟ = log m. n→∞ n2
x→a ⎝ x→a ⎠
lim g ( x) According to Sandwich Theorem or Squeeze Principle
(b) lim e g ( x ) = e x→a
= em.
x→a ( x ) + ( 2 x ) + ... + ( nx )
n lim =0
⎡ ⎤ n→∞ n2
7. lim[ f ( x )]n = ⎢ lim f ( x ) ⎥ = ln, for all n ∈ N.
x→a ⎣ x → a ⎦
8. Sandwich Theorem (or Squeeze Principle). CAUTION
If f, g and h are functions such that f (x) ≤ g (x) ≤ h (x)
for all x in some neighbourhood of the point a (except The converse of the above result may not be true,
possibly at x = a) and if lim f ( x ) = l = lim h ( x ) , i.e., lim f( x ) = |l| = |l| ⇒ lim f( x ) = l
x→a x→a x →a x →a
then lim g ( x ) = l.
x→a

9. If lim f ( x ) = l, then lim f ( x ) = |l |.


x→a x→a
EVALUATION OF LIMITS
I M P O R TA N T P O I N T S The problems on limits can be divided into the following
categories:
Sandwich theorem helps in calculating the limits, when limits
cannot be calculated using the usual method Limits

SOLVED EXAMPLE Algebraic Trigonometric Exponential and


limits limits logarithmic limits
( x ) + ( 2 x ) + (3 x ) + ... + ( nx )
3. lim , where
n→∞ n2
{x} = x – [x] denotes the fractional part of x, is
ALGEBRAIC LIMITS
(A) 1 (B) 0
1 The following methods are useful for evaluating limits of
(C) (D) None of these algebraic functions:
2
11.4 Chapter 11

Method of Factorization Solution: (A)


⎡ x -3 ⎤
If f (x) and g (x) are polynomials and g (a) ≠ 0, then we have lim ⎢log a ⎥
x →3 ⎣ x + 6 - 3⎦
lim f ( x )
f ( x) x→a f ( a)
lim = = . ⎡ ( x - 3) ( x + 6 + 3) ⎤
x→a g ( x) lim g ( x ) g ( a) = lim ⎢log a ⎥
x→a x →3
⎣ ( x - 3) ⎦
Now, if f (a) = 0 = g (a), then (x – a) is a factor of both f (x)
and g (x). We cancel this common factor (x – a) from both = lim log a ( x + 6 + 3) = loga 6
x →3
the numerator and denominator and again put x = a in the
given expression. If we get a meaningful number then that Standard Formula
number is the limit of the given expression, otherwise we
x n - an
repeat this process till we get a meaningful number. lim = nan – 1,
x→a x-a

SOLVED EXAMPLE where n ∈ Q, the set of rational numbers.

3
x2 - 2 3 x + 1 SOLVED EXAMPLE
4. lim is equal to
x →1 ( x - 1) 2
x r - 1r
n
1 1 6. lim ∑ =
r =1 x - 1
(A) (B) x →1
9 6 n( n + 1)
(A) 0 (B)
1 2
(C) (D) None of these
3 (C) 1 (D) None of these
Solution: (A) Solution: (B)
3
n
x r - 1r
lim
x2 - 2 3 x + 1
= lim
y2 - 2 y + 1 We have, lim ∑
r =1 x - 1
x →1
x →1 ( x - 1) 2 y →1 ( y 3 - 1) 2
n
n( n + 1)
[Putting 3
x = y; as x → 1, y → 1] = ∑ r ⋅1r - 1 =1+2+3+…+n=
2
.
r =1
( y - 1) 2
= lim
y →1 ( y - 1) 2 ( y 2 + y + 1) 2 LIMIT OF AN ALGEBRAIC FUNCTION
1 1 WHEN X → ∞
= lim = .
y →1 2
( y + y + 1) 2 9 f ( x)
In order to find the limit of a function of the type as
g ( x)
Method of Rationalization x → ∞, where f (x) and g (x) are algebraic functions of x, it
is convenient to divide all the terms of f (x) and g (x) by the
This method is useful where radical signs (i.e., expressions
highest power of x in numerator and denominator both and
of the form a ± b ) are involved either in the numera-
use the following standard limits:
tor or in the denominator or both. The numerator or (and)
1
the denominator (as required) is (are) rationalised and limit 1. lim =0
x →∞ x
taken after cancelling out the common factors.
1
2. lim p = 0, if p > 0.
x →∞ x
SOLVED EXAMPLE
TRICK(S) FOR PROBLEM SOLVING
⎛ x -3 ⎞
5. The value of lim ⎜ log a ⎟ is
x →3 ⎝ x + 6 - 3⎠ ⎧ ∞, if a > 1
⎪ if a = 1
(A) loga 6 ⎪ 1,
 lim a n = ⎨
(B) loga 3 n→∞ ⎪ 0, if - 1 < a < 1
(C) loga 2 ⎪⎩does not exist, if a ≤ - 1
(D) None of these
Limits 11.5

1 n 3
a0 x + a1x
p p -1
+ … … + ap - 1 x + ap
\ S= ∑ [r - 2nr 2 + (n2 - 2n - 1)r + n2 ]
2 r =1
 lim q -1
x→∞ b0 x + b1x
q
+ … … + bq - 1 x + bq 2
1 ⎡ ⎧ n( n + 1) ⎫ ⎧1 ⎫
⎧ a0
⇒ S= ⎢ ⎨ 2 ⎬ - 2n ⎨ 6 n ( n + 1)( 2n + 1)⎬
⎪b , if p = q 2 ⎣⎩ ⎭ ⎩ ⎭
= ⎪⎪0,
0
⎧1 ⎫ ⎤
⎨ if p < q + ( n2 - 2n - 1) ⎨ n( n + 1)⎬ + n2 ( n) ⎥
⎪∞, if p > q ⎩ 2 ⎭ ⎦

⎪⎩ Solving and rearranging, we have
1 4
S= ( n - 11n3 - 19n2 + 6 n)
24
Some Useful Summations S 1 ⎛ n4 - 11n3 - 19n2 + 6 n ⎞
\ lim 4 = lim ⎜ ⎟
n ( n + 1) n→∞ n n → ∞ 24 ⎝ n4 ⎠
1. S n = 1 + 2 + 3 + … + n =
2 1 ⎛ 11 19 6 ⎞
n ( n + 1) ( 2n + 1) = lim ⎜1 - - 2 + 3 ⎟
24 n → ∞ ⎝ n ⎠
2 2 2 2 2
2. S n = 1 + 2 + 3 + … + n = n n
6
n -1 ⎧ n-r ⎫⎪
⎡ n ( n + 1) ⎤
2 1 ⎪ 1
3. S n3 = 13 + 23 + 33 + … + n3 = ⎢ ⎥
\ lim
n→∞ 4 ∑ ⎨⎪(r + 1) ∑ k ⎬⎪ =
24
⎣ 2 ⎦ n r =0 ⎩ k =1 ⎭
n
a (1 - r )
4. S arn – 1 = a + ar + ar2 + … + arn – 1 = ; n⎛ r 3 - 1⎞
provided r < 1. 1- r 8. lim ∏⎜ 3 ⎟
r = 3 ⎝ r + 1⎠
n→∞

1 6
SOLVED EXAMPLES (A) (B)
3 7
2
7. The value of (C) - (D) None of these
3
1 ⎡ ⎛ n ⎞ ⎛ n -1 ⎞ ⎛n-2 ⎞ ⎤ Solution: (B)
lim ⎢1
n → ∞ n4 ⎢ ⎝
⎜ ∑ k ⎟ + 2 ⎜ ∑ k ⎟ + 3 ⎜ ∑ k ⎟ + ... + n ⋅ 1⎥
⎥⎦ (n – 2)th factor of the series is
⎣ k =1 ⎠ ⎝ k =1 ⎠ ⎝ k =1 ⎠
will be n - 1 n2 + n + 1
tn = ⋅
1 1 1 1 n + 1 n2 - n + 1
(A) (B) (C) (D)
24 12 6 3 Therefore, required limit = lim t3t 4 t5 ... t n - 2 t n - 1t n
n→∞
Solution: (A) ⎡⎛ 2 3 4 n - 3 n - 2 n - 1⎞
= lim ⎢⎜ ⋅ ⋅ ... ⋅ ⋅
n + 1⎟⎠
The (r + 1)th term of the series is n→∞ ⎝ 4 5 6
n-r ⎣ n -1 n
tr + 1 = ( r + 1) ∑ k
⎛ 13 ⎞ 21 31 n + n +1 ⎤
2
k =1 ⋅ ⎜ ⎟ ⋅ ⋅ ... 2 ⎥
⎝ 7 ⎠ 13 21 n - n + 1⎦
⇒ tr + 1 = (r + 1)[1 + 2 + 3 + … (n – r) terms]
1 2 ⋅ 3 n2 + n + 1 6
⇒ tr + 1 = ( r + 1) ( n - r )( n - r + 1) = lim ⋅ = .
n → ∞ n( n + 1) 7 7
2
1 9. If [x] denotes the integral part of x, then
⇒ tr + 1 = ( r + 1)( n2 - rn + n - rn + r 2 - r )
2
1 ⎛ n 2 ⎞
⎜ ∑ [k x ]⎟ =
1 lim
⇒ tr + 1 = ( r + 1)( r 2 - (1 + 2n)r + n2 ) n→∞ n3 ⎝
2 k =1 ⎠
1 3 x
⇒ tr + 1 = ( r - 2nr 2 + ( n2 - 2n - 1)r + n2 ) (A) 0 (B)
2 2
n -1 x x
Now, S= ∑ tr + 1 (C)
3
(D)
6
r=0
11.6 Chapter 11

Solution: (C) Solution: (C)


1 ⎛ n ⎞ 5r + 2 r
n
L = lim 3 ⎜ ∑ [k 2 x ]⎟
n→∞ n ⎝
Required limit = lim
n→∞
∑ 10 r
k =1 ⎠ r =1

Since k2x – 1 ≤ (k2x) < k2x ⎧⎪⎛ 1 ⎞ r ⎛ 1 ⎞ r ⎫⎪


n

n n n
= lim
n→∞
∑ ⎨⎜⎝ 2 ⎟⎠ + ⎜⎝ 5 ⎟⎠ ⎬
r =1⎩⎪ ⎭⎪
⇒ ∑ (k 2 x - 1) ≤ ∑ (k 2 x) < ∑ k 2 x
k =1 k =1 k =1 ⎧ ⎛ 1⎞ ⎫
n n
⎛ 1⎞
⎛ n ⎞ n n ⎛ n ⎞ ⎪ 1 - ⎜⎝ ⎟⎠ 1 - ⎜⎝ ⎟⎠ ⎪
⎪1 2 1 5 ⎪
⇒ x ⎜ ∑ k 2 ⎟ - ∑ (1) ≤ ∑ [k 2 x ] < x ⎜ ∑ k 2 ⎟ = lim ⎨ ⋅ + ⎬
⎝ k =1 ⎠ k =1 k =1 ⎝ k =1 ⎠ n→∞ 2
⎪ 1 5 1 ⎪
1- 1-
⎪ 2 5 ⎪
xn( n + 1)( 2n + 1) ⎩ ⎭
⇒ -n
6 1 5
= 1+ =
n
xn ( n + 1)( 2n + 1) 4 4
≤ ∑ [k 2 x ] <
6
k =1 2x - 3 2 x 2 + 5x
3 11. lim f ( x ), where < f (x) < , is
Dividing throughout by n , we have x →∞ x x2
xn ( n + 1)( 2n + 1) 1 (A) 1 (B) 2 (C) –1 (D) –2
3
-
6n n2
Solution: (B)
n
[k 2 x ] xn ( n + 1)( 2n + 1)
≤ ∑ n3
<
6 n3 lim
2x - 3 ⎛ 3⎞
= lim ⎜ 2 - ⎟ = 2
k =1 x →∞ x x →∞ ⎝ x⎠
x⎛ 1⎞ ⎛ 1⎞ 1 n
[k 2 x ] 2 x 2 + 5x ⎛ 5⎞
⇒ ⎜1 +
6⎝
⎟ ⎜2 +
n⎠ ⎝
⎟⎠ - 2 ≤ ∑ 3 and lim = lim ⎜ 2 + ⎟ = 2,
n n k =1 n x →∞ x 2 x →∞ ⎝ x⎠
x⎛ 1⎞ ⎛ 1⎞ \ Using Sandwitch theorem, lim f ( x ) = 2.
< ⎜1 + ⎟⎠ ⎜⎝ 2 + ⎟⎠ x →∞
6⎝ n n
Taking limits as n → ∞, we get TRIGONOMETRIC LIMITS
⎡x ⎛ 1⎞ ⎛ 1⎞ 1⎤ For finding the limits of trigonometric functions, we use
lim ⎢ ⎜1 + ⎟ ⎜ 2 + ⎟ - 2 ⎥ ≤ L
n→∞ ⎣6 ⎝ n⎠ ⎝ n⎠ n ⎦ trigonometric transformations and simplify. The following
results are quite useful:
x⎛ 1⎞ ⎛ 1⎞
< lim ⎜ 1+ ⎟ ⎜2 + ⎟ sin x
n→∞ 6 ⎝ n⎠ ⎝ n⎠ 1. (a) lim =1
x →0 x
1
Since, as n → ∞, we have → 0 (b) lim cos x = 1
n x →0
x x tan x
⇒ ≤L< (c) lim =1
3 3 x →0 x
sin -1 x
According to Squeeze Principle or Sandwich Theorem, (d) lim =1
we have x→0 x
x tan -1 x
L= . (e) lim =1
3 x→0 x
⎧⎪ 7 sin x 0 p
29 133 5n + 2n ⎫⎪ (f) lim = .
10. lim ⎨ + 2 + 3 + ... + ⎬ is equal to x →0 x 180
n → ∞ ⎪10
⎩ 10 10 10 n ⎭⎪
2. lim f ( x ) = lim f ( a + h) , where a ≠ 0, on taking
3 5 1 x→a h→ 0
(A) (B) 2 (C) (D) x = a + h.
4 4 2
Limits 11.7

Some Useful Expansions x


sin 2
1
= lim ⋅ 2 ⋅ 1 + cos x + cos x
x3 x5 x7 x→0 2 x x
1. sin x = x - + - + ... ∞; x ∈  cos ⋅ cos x
3! 5! 7 ! 2 2
x 2 x 4 x6 1 3
= ⋅1⋅ =
3
2. cos x= 1 - + - + ... ∞; x ∈ 
2! 4 ! 6 ! 2 1 2
x3 2 17 7 62 9 ⎛ x x x x⎞
3. tan x = x + + x3 + x + x + ... ∞ ; 13. lim ⎜ cos cos cos … cos n ⎟ =
3 15 315 2835 n→∞ ⎝ 2 4 8 2 ⎠
p
|x| < x sin x
2 (A) (B)
x2 5 4 61 6 p sin x x
4. sec x = 1 + + x + x + ... ∞; 0 < |x| < (C) 0 (D) None of these
2 24 720 2
1 x 7 3 31 5 Solution: (B)
5. cosec x = + + x + x + ... ∞; 0 < |x| < p
x 6 360 15120 x x x x
Required Limit = lim cos cos 2 cos 3 ... cos n
3 n→∞ 2
1 x x 2 5 2 2 2
6. cot x = - - - x - ... ∞; 0 < |x| < p
x 3 45 945 1 ⎡ x x ⎛ x x ⎞⎤
= lim cos ... cos n - 1 ⎜ 2 sin n cos n ⎟ ⎥
1 x 3 1.3 x 5 1. 3 . 5 x 7 n→∞ x ⎢⎣ 2 2 ⎝ 2 2 ⎠⎦
7. sin– 1 x = x + + + + ... ∞; |x| < 1 2 sin n
2 3 2.4 5 2 . 4 . 6 . 7 2
p ⎡ x ⎛ x ⎞⎤
8. cos– 1 x = - sin -1 x = lim
1 x
2 n→∞ x ⎢cos 2 ... ⎜⎝ 2 cos n - 1 sin n - 1 ⎟⎠ ⎥
22 sin n ⎣ 2 2 ⎦
p ⎧⎪ 1 x 3 1.3 x 5 1.3.5 x 7 ⎪⎫ 2
= - ⎨x + + ⋅ + ⋅ + ... ∞⎬; |x| < 1
2 ⎪⎩ 2 3 2.4 5 2.4.6 7 ⎭⎪ …………………………………………......................
…………………………………………......................
9. tan–1x =
⎧ x3 x5 x7
1 ⎛ x x⎞ sin x
= lim ⎜⎝ 2 cos sin ⎟⎠ = nlim
⎪ x- + - + ... ∞; | x |<1 n→∞ x 2 2 → ∞ ⎛ x⎞
⎪ 3 5 7 2n sin n 2n sin ⎜ n ⎟
⎨ 2 ⎝2 ⎠
⎪± p - 1 + 1 - 1 + 1 - ... ∞; ⎧ + if x ≥ 1
⎪⎩ 2 x 3x 3 5 x 5 7 x 7 ⎨ ⎧ ⎛ x⎞ ⎫
⎩ - if x ≤ - 1
sin x ⎪⎪ ⎜⎝ n ⎟⎠ ⎪⎪ sin x
2
= lim ⎨ ⎬ = .
x n→∞⎪ ⎛ x ⎞ ⎪ x
SOLVED EXAMPLES sin ⎜ n ⎟
⎪⎩ ⎝ 2 ⎠ ⎪⎭
1 - cos3 x
12. lim = q 1 q q⎞
x → 0 x sin x cos x ⎛ 1 1
14. lim ⎜ tan q + tan + 2 tan 2 + … + n tan n ⎟ =
3 1 n→∞ ⎝ 2 2 2 2 2 2 ⎠
(A)
2 2 1 1
(A) (B) - 2 cot 2q
(C) 1 (D) None of these q q
(C) 2 cot 2q (D) None of these
Solution: (A)
1 - cos3 x Solution: (B)
lim
x → 0 x sin x cos x sin 2q
tan 2q =
3
(1 - cos x ) (1 + cos x + cos x ) cos 2q
= lim
x→0 x sin x cos x 1 2 sin q cos q
=
2 x cot 2q cos 2 q - sin 2 q
2 sin (1 + cos 2 x + cos x )
= lim 2
x x cos 2 q - sin 2 q
x→0
2 x sin cos cos x ⇒ cot 2q =
2 2 2 sin q cos q
11.8 Chapter 11

cos 2 q sin 2 q 4 4 2 3
⇒ 2cot 2q = - (A) (B) – (C) (D)
sin q cos q sin q cos q 3 3 3 4
⇒ 2cot 2q = cot q – tan q Solution: (A)
⇒ tan q = cot q – 2 cot 2q (1) ⎡ ⎛p ⎞ ⎛p ⎞⎤
2 ⎢ 3 sin ⎜ + h⎟ - cos ⎜ + h⎟ ⎥
Now, tan q = cot q – 2 cot 2q ⎝ ⎠ ⎝ ⎠⎦
lim ⎣
6 6
1 q 1 q h→ 0 3h ( 3 cos h - sin h)
⇒ tan = cot - cot q
2 2 2 2
⎡ ⎛1 3 ⎞ ⎛ 3 1 ⎞⎤
1 q 1 q 1 2 ⎢ 3 ⎜ cos h + sin h⎟ - ⎜ cos h - sin h⎟ ⎥
⇒ tan 2 = cot - cot q ⎢ ⎝2 2 ⎠ ⎝ 2 2 ⎠ ⎥⎦
2 2
2 2 2 2 2 = lim ⎣
h→ 0 3h( 3 cos h - sin h)
1 q 1 q 1 q
⇒ n
tan n
= n cot n - n - 1 cot n - 1 2 [ 2 sin h]
2 2 2 2 2 2 = lim
1 q 3 h ( 3 cos h - sin h)
h→ 0
⇒ S = - 2 cot 2q + n cot n
2 2 sin h
4⋅
Therefore, h
= lim
q 1 q q⎞ h→ 0 3 ( 3 cos h - sin h)
⎛ 1 1
lim ⎜ tan q + tan + 2 tan 2 + ... + n tan n ⎟
n→∞⎝ 2 2 2 2 2 2 ⎠ 4 4
= =
⎛ 1 q⎞ 3 ( 3 - 0) 3
= lim S = lim ⎜ - 2 cot 2q + n cot n ⎟
n→∞ n→∞⎝ 2 2 ⎠ x n - sin x n
⎛ q ⎞ 17. If lim is non-zero finite, then n may be
x → 0 x - sin n x
1⎜ n ⎟
= - 2 cot 2q + lim ⎜ 2 ⎟ equal to
n→∞q q
⎜ tan ⎟ (A) 1 (B) 2
⎝ 2n ⎠ (C) 3 (D) None of these
1
= - 2 cot 2q + Solution: (A)
q
tan ([ - p 2 ]x ) - x 2 tan ([ - p 2 ]) x - sin x
15. lim equals, where [ ] Clearly, for n = 1, lim = 1.
x→0 x - sin x
x→0 sin 2 x
denotes the greatest integer function ⎛ cot 4 x ⎞
18. lim ⎜ cosec3 x ⋅ cot x - 2 cot 3 x ⋅ cosec x + is
(A) 0 (B) 1 x →0 ⎝ sec x ⎟⎠
(C) tan 10 – 10 (D) ∞
equal to
Solution: (C) (A) 1 (B) – 1
Since, [–p2] = [–9.87] = –10, therefore (C) 0 (D) None of these
tan[ - p 2 ]x 2 - x 2 tan[ - p 2 ] Solution: (A)
lim 2
x→0 sin x ⎡ cot 4 x ⎤
2 2 2 lim ⎢cosec3 x ⋅ cot x - 2 cot 3 x ⋅ cosec x + ⎥
tan ( - 10 x ) - x tan ( - 10) x x →0
⎣ sec x ⎦
= lim 2
x→0 x sin 2 x ⎛ cos x 2 cos3 x cos5 x ⎞
tan (10 x 2 ) x 2 tan 10 = lim ⎜ 4 - + ⎟
= – lim + lim x → 0 ⎝ sin x sin 4 x sin 4 x ⎠
x→0 x2 x→0 x2
2 cos x (1 - cos 2 x ) 2
tan (10 x ) = lim = lim cos x = 1
= lim ( - 10) + tan 10 x →0 sin 4 x x →0
x→0 10 x 2
= tan 10 – 10
EXPONENTIAL AND LOGARITHMIC LIMITS
⎡ ⎛p ⎞ ⎛p ⎞⎤
2 ⎢ 3 sin ⎜ + h⎟ - cos ⎜ + h⎟ ⎥ For finding the limits of exponential and logarithmic func-
⎝ ⎠ ⎝ ⎠⎦
16. lim ⎣
6 6 tions, the following results are useful:
is equal to
h→ 0 3h ( 3 cos h - sin h)
Limits 11.9

ex - 1 ⎛ ⎞
1. lim =1 x 2 x 4 x6 x2
x→0 x ⎜ 1 - + - + ...⎟ - 1 +
⎝ 2! 4 ! 6 ! ⎠ 2
ax - 1 = lim
2. lim = loge a, a > 0 x→0 x4
x→0 x
⎛1 ⎞
ax - bx ⎛ a⎞ = lim ⎜ + terms containing x and its powers⎟
3. lim = loge ⎜ ⎟ ; a, b > 0 x → 0 ⎝ 4! ⎠
x→0 x ⎝ b⎠
n
(1 + x ) - 1 1 1
4. lim =n = =
x→0 x 4! 24
n
⎛ 1⎞
5. lim (1 + x )1/ x = lim ⎜1 + ⎟ = e
x→0 n →∞ ⎝ n⎠ CAUTION
6. lim (1 + a h)1/ h = ea If [⋅] denotes the greatest integer function, then
h→ 0
log x lim [ - x ] = [0] = 0
7. lim = 0, (m > 0) x →0
x →∞xm Is the above statement true?
log a (1 + x ) No. If fact, lim [ - x ] = lim [0 - x ] = lim - 1 = –1
8. lim = loga e, (a > 0, a ≠ 1) x →0 x →0 x→ 0
x→0 x
Thus, limit must be applied only after removing [⋅] sign.
x
⎛ a⎞
9. lim ⎜1 + ⎟ = ea
x →∞ ⎝ x⎠
⎛ 1 ⎞
f ( x) SOLVED EXAMPLES
10. lim ⎜1 + = e, where f (x) → ∞ as x → ∞.
x →∞ ⎝ f ( x ) ⎟⎠ x sin{x}
19. lim , where {x} denotes the fractional part of x,
x →1 x -1
11. lim (1 + f ( x )1/ f ( x ) = e.
x→a is equal to
(A) –1 (B) 0
Some Useful Expansions (C) 1 (D) Does not exist
x x 2 x3 Solution: (D)
1. ex = 1 + + + + ... to ∞
1! 2 ! 3! x sin{x}
LHL = lim
x → 1- x - 1
x x 2 x3
2. e–x = 1 – + - + ... to ∞ Let x = 1 – h, as x → 1, h → 0
1! 2 ! 3!
(1 - h) sin{1 - h}
x 2 x3 ⇒ LHL = lim
3. loge (1 + x) = x – + - ... to ∞, – 1 < x ≤ 1 h→0 h
2 3
(1 - h) sin (1 - h)
x 2 x3 ⇒ LHL = lim
4. loge (1 – x) = – x – - - ... to ∞, – 1 ≤ x < 1 h→0 h
2 3 (1 - h)
( x log a) 2 \ LHL = lim sin (1) = ∞
h→0 h
5. ax = ex log a = 1 + x log a + + ... to ∞
2! x sin ( x )
n ( n - 1) 2 Now, RHL = lim
6. (1 + x)n = 1 + nx + x + ... to ∞, – 1 < x < 1, x → 1+ x - 1
2!
n being any negative integer or fraction. Let x = 1 + h, as x → 1, h → 0
(1 + h) sin (1 + h)
The expansion formulae mentioned above can be used with ⇒ RHL = lim
h→0 h
advantage in simplification and evaluation of limits.
(1 + h) sin h
⇒ RHL = lim = lim (1 + h)
x2 h→0 h h→0
cos x - 1 +
For example, lim 2 \ RHL = (1 + 0) = 1
x→0 x4
Since LHL ≠ RHL,
\ the limit of the function Does not exist at x = 1.
11.10 Chapter 11

⎛ b⎞ ⎡ x ⎤ where [x] denotes the greatest integer less than or


20. lim ⎜ ⎟ ⎢ ⎥ where a > 0, b > 0 and [x] denotes equal to x, then lim f ( x ) is equal to
x →0 ⎝ x⎠ ⎣a⎦
+
x →0
greatest integer less than or equal to x is 1
(A) – (B) 1
1 b 2
(A) (B) b (C) (D) 0 p
a a (C) (D) Does not exist
Solution: (D) 4

⎛ b⎞ ⎡ x ⎤ ⎛ b ⎞ ⎡0 + h⎤ Solution: (D)
lim ⎜ ⎟ ⎢ ⎥ = lim ⎜ ⎟ tan -1 ([ - h] - h)
x →0 ⎝ x⎠ ⎣a⎦
+
h → 0 ⎝ 0 + h⎠ ⎢ ⎥
⎣ a ⎦ LHL = lim f (0 - h) = lim
h→ 0 h→ 0 [ - h] + 2 h
⎛ b⎞ ⎡ h⎤ tan -1 ( -1 - h)
= lim ⎜ ⎟ ⎢ ⎥ = 0 = lim
h → 0 ⎝ h⎠ ⎣ a ⎦
h→ 0 ( 2h - 1)
⎧ sin[ x ]
, [ x] ≠ 0 tan -1 (1 + h)
⎪ = lim
21. If f (x) = ⎨ [ x ] , where [x] denotes the h→ 0 (1 - 2h)
⎪0 , [ x] = 0
⎩ p /4 p
greatest integer ≤ x, then lim f ( x ) equals = = .
x →0 1 4
tan -1 ([h] + h)
(A) 0 (B) –1 RHL = lim f (0 + h) = lim
(C) 1 (D) None of these h→ 0 h→ 0 [h] - 2h
tan -1 ( h) 1
Solution: (A) = lim =–
h→ 0 -2h 2
sin[ - h] sin( -1)
lim f (0 - h) = lim = lim = sin 1. Since LHL ≠ RHL
h→ 0 h→ 0 [ - h] h→ 0 ( -1)
\ lim f ( x ) Does not exist.
sin[h] x →0
lim f (0 + h) = lim
h→ 0 h→0 [ h]
TRICK(S) FOR PROBLEM SOLVING
=1 [∵ h → 0 ⇒ (h) → 0]
\ lim f ( x ) does not exist.  If lim f( x ) = A > 0 and lim g( x ) = B, then
x →0 x →a x →a

lim [f( x )]g ( x ) = AB


22. Let f (x) = x – [x], where [x] denotes the greatest integer x →a

≤ x and g (x) = lim


[ f ( x)]2n - 1 , then g (x) =  If lim f( x ) = 1 and lim g( x ) = ∞, then
x →a x →a

[ ]
n→∞ f ( x ) 2 n + 1
lim [f( x )]g( x ) = e
lim
x→a
g( x )[ f ( x )-1]

x →a
(A) 0 (B) 1
(C) –1 (D) None of these
Solution: (C)
SOLVED EXAMPLES
As 0 ≤ x – [x] < 1 ∀ x ∈ R, 0 ≤ f (x) < 1.
\ lim [ f ( x ) ]
2n
=0 729 x - 243x - 81x + 9 x + 3x - 1
n→∞
24. If lim 3
= k(log 3)3,
x→0 x
Thus, forx ∈ R, g (x) = lim
[ f ( x)]2n - 1 then k =
[ ]
n→∞ f ( x ) 2 n + 1 (A) 4 (B) 5
(C) 6 (D) None of these
0 -1
= = –1 Solution: (C)
0 +1
Required limit
⎧ tan -1 ([ x ] + x ) 243x (3x - 1) - 9 x (32 x - 1) + (3x - 1)
⎪ , [ x] ≠ 0 = lim
23. If f (x) = ⎨ [ x ] - 2 x x→0 x3
⎪0 , [ x] = 0 (3 - 1) {( 243) - ( 27) x - 9 x + 1}
x x
⎩ = lim
x→0 x3
Limits 11.11

(3x - 1) {( 243) x - ( 27) x - 9 x + 1} ⎛ 1 + tan x ⎞


1/sin x
= lim 27. lim ⎜ is equal to
x→0 x3 ⎟
x → 0 ⎝ 1 + sin x ⎠
(3 - 1) (9 - 1) ( 27 x - 1)
x x
(A) 0 (B) 1
= lim
x→0 x x x (C) –1 (D) None of these
= log 3 ⋅ log 9 ⋅ log 27 Solution: (B)
= log 3 ⋅ 2log 3 ⋅ 3log 3 1 + tan x
Let f (x) =
= 6(log 3) = k(log 3)3 3
(given) 1 + sin x
\ k=6 1
and g (x) = .
sin x
25. If a and b be the roots of ax2 + bx + c = 0, then Clearly f (x) → 1 and g (x) → ∞ as x → 0.
lim (1 + ax 2 + bx + c)1 ( x -a ) is 1 /sin x 1 ⎛ 1+ tan x ⎞
- 1⎟
x→a ⎛ 1 + tan x ⎞ lim ⎜
sin x ⎝ 1+ sin x ⎠
\ = e
x →0

(A) log |a (a – b )| (B) ea (a – b) lim ⎜ ⎟


x → 0 ⎝ 1 + sin x ⎠

(C) ea (b – a)
Solution: (B)
(D) None of these
{Using lim [ f ( x)]
x→a
g( x)
=e
lim g ( x ) [ f ( x ) -1]
x→a
}
1
lim ⎡(1+ ax 2 + bx + c ) -1⎤⎦ 1 ⎛ tan x - sin x ⎞ 1- cos x
( x -a ) ⎣
lim (1 + ax 2 + bx + c)1 ( x -a ) = e
x →a
lim ⎜ ⎟ lim
sin x ⎝ 1+ sin x ⎠ cos x (1+ sin x )
= e0 = 1
x →0
= e = e
x →0

x →a
lim g ( x )[ f ( x ) -1]
[Using lim [f (x)]g (x) = e →
x a
tan
px
x→a ⎛ x⎞ 2a
provided f (x) → 1 and g (x) → ∞ as x → a] 28. lim ⎜ 2 - ⎟ is equal to
x→a ⎝ a⎠
( ax + bx + c )
2
a ( x -a )( x - b )
lim lim
= e
x →a
( x -a )
= e
x →a
( x -a ) (A) ep/2 (B) e2/p (C) e–2/p (D) e–p/2

[∵ a, b are roots of ax2 + bx + c = 0] Solution: (B)


px ⎛ p x⎞ ⎛ x ⎞
= ea (a – b)
tan ⋅ 2 - -1
⎛ x⎞ 2a lim tan ⎜
⎝ 2 a ⎟⎠ ⎜⎝ a ⎟⎠
lim ⎜ 2 - ⎟ = e
x→a

1 x→a ⎝ a⎠
⎛ sin x ⎞ x - a
26. lim ⎜ ⎟
x → a ⎝ sin a ⎠
, a ≠ np, n is an integer, equals
{Using lim [ f ( x)]x→a
g( x)
=e
lim g ( x ) [ f ( x ) -1]
x→a

(A) ecot a (B) etan a (C) esin a (D) ecos a ⎫


as f ( x ) → 1 and g ( x ) → ∞ as x → a⎬
Solution: (A) ⎭
⎛ x⎞ ⎛ p x⎞ (1- x a )
1 1 lim ⎜1- ⎠⎟ tan ⎝⎜
x→a ⎝
⎟ lim
2a ⎠ →
cot (p x 2 a )
= e a
= e
x a

⎛ sin x ⎞ x -a ⎛ sin x - sin a ⎞ x -a


lim ⎜ = lim ⎜1 +
⎝ sin a ⎟⎠ ⎟ -1 / a
x→a x→a ⎝ sin a ⎠ lim
x→a
⎛ p x⎞ p 2 2 ⎛ p x⎞
- cosec 2 ⎜ ⎟ ⋅ lim sin ⎜
⎝ 2 a ⎟⎠
sin x - sin a
= e ⎝ 2a ⎠ 2a
= e p
= e2/p
x→a

⎡ sin a ⎤ ( x - a )sin a
⎢ ⎧ ⎛ sin x - sin a ⎞ ⎫ sin x - sin a ⎥ a
= lim ⎢ ⎨1 + ⎜ ⎟⎬ ⎥ 29. lim (cos x + a sin bx ) x is equal to
x→a ⎩ ⎝ sin a ⎠ ⎭ x →0
⎢⎣ ⎥⎦
(A) e - a b (D) e - b a
2 2 2 2

(B) e ab (C) e a b
sin x - sin a
= lim Solution: (C)
e ( x - a) sin a
x→a
a a
lim (cos x + a sin bx -1)
2 ⎛ x + a⎞ ⎛ x - a⎞ 1 lim (cos x + a sin bx ) x = x→ 0
e x
= lim cos ⎜ ⎟⎠ sin ⎜⎝ ⎟⋅ x →0
e x→a
x-a ⎝ 2 2 ⎠ sin a ⎡ Using lim f ( x ) f ( x ) = e lim f ( x )[ f ( x ) -1]
⎣⎢ x→a
[ ] x→a

⎛ x + a⎞ ⎡ ⎛ x - a⎞ ⎛ x - a⎞ ⎤ 1
= lim cos ⎜ ⎢sin ⎜⎝ 2 ⎟⎠ ⎜⎝ ⎟
e ⎝ 2 ⎟⎠
x→a
⎣ 2 ⎠ ⎥⎦ sin a as f (x) → 1 and f (x) → ∞ as x → a]
cos a a ( - sin x + ab cos bx )
lim 2

= e sin a = ecot a = e x →0
1 = ea b
11.12 Chapter 11

2
sin x ⎛ ⎛ x ⎞⎞
⎛ sin x ⎞ x - sin x ⎜ sin ⎝⎜ 2 n ⎠⎟ ⎟ x 2
30. The value of lim ⎜ ⎟ is -2 lim ⎜ ⎟ ⋅ 2 ⋅n
x →0 ⎝ x ⎠ ⎜ x ⎟ 4n
n→∞

⎜⎝ 2 n ⎟⎠
(A) 1 (B) –1 =e
(C) 0 (D) None of these x2
-2 × lim
Solution: (B) = e n→∞
4n = e0 = 1
sin x sin x ⎛ sin x ⎞
-1⎟
⎛ sin x ⎞ x - sin x lim ⎜
EVALUATION OF LIMITS USING
= e x - sin x ⎝ ⎠
x →0
x
lim ⎜ ⎟
x →0 ⎝ x ⎠ L’HOSPITAL’S RULE
⎡ Using lim f ( x ) g ( x ) = e lim g ( x )[ f ( x ) -1]
⎢⎣ x→a
[ ] x→a Besides the methods given above to evaluate limits, there
is yet another method for finding limits, usually known as
sin x L’Hospital’s Rule as given below for indeterminate forms:
as f ( x) = → 1 and g ( x )
x
⎛ 0⎞
sin x 1. ⎜ ⎟ form: If lim f ( x ) = 0 and lim g ( x ) = 0, then
sin x x ⎝ 0⎠ x→a x→a
= = →∞ as x → 0⎤⎦ f ( x) f ′ ( x)
x - sin x sin x lim = lim , provided the limit on the
1- x → a g ( x) x → a g ′ ( x)
x
sin x
lim - R.H.S. exists.
= e x→ 0
x = e– 1 Here, f ′ is derivative of f.
1- cos( x +1)
⎛ ∞⎞
⎛ x 4 + x 2 + x + 1⎞ ( x +1) 2 2. ⎜ ⎟ form: If lim f ( x ) = ∞ and lim g ( x ) = ∞,
31. lim ⎜ ⎟ is equal to ⎝ ∞⎠ x→a x→a
x →-1 ⎝ x2 - x + 1 ⎠ f ( x) f ′ ( x)
then lim = lim , provided the limit on
1/ 2 1/ 2 x → a g ( x) x → a g ′ ( x)
⎛ 2⎞ ⎛ 3⎞
(A) 1 (B) ⎜ ⎟ (C) ⎜ ⎟ (D) e1/2 the R.H.S. exists.
⎝ 3⎠ ⎝ 2⎠
Solution: (B) Note that sometimes we have to repeat the process if the
1- cos( x +1) 0 ∞
form is or again.
⎛ x 4 + x 2 + x + 1⎞ ( x +1) 2 0 ∞
lim ⎜ ⎟
x →-1 ⎝ x2 - x + 1 ⎠
⎛ x +1⎞ TRICK(S) FOR PROBLEM SOLVING
2 sin 2 ⎜
⎝ 2 ⎟⎠
⎛ x4 + x2 + x + 1⎞ ( x + 1) 2 f( x )
= lim ⎜ ⎟
 L’Hospital’s Rule is applicable only when
g( x )
becomes of
x →-1 ⎝ x2 - x + 1 ⎠ the form
0
or

.
⎛ ⎛ x +1⎞ ⎞
2 0 ∞
sin
1 ⎜ ⎜⎝ 2 ⎟⎠ ⎟
0 ∞
⎜ ⎟  If the form is not or , simplify the given expression till
⎛ x4 + x2 + x + 1⎞ ⎜⎜ ⎛⎜ x +1⎞⎟ ⎟⎟
2 ⎛ 2⎞
1/ 2 0 ∞
= lim ⎜ ⎟ ⎝ ⎝ 2 ⎠ ⎠ = ⎜ ⎟ 0 ∞
x →-1 ⎝ x2 - x + 1 ⎠ ⎝ 3⎠ it reduces to the form or and then use L’Hospital’s
rule. 0 ∞
n For applying L’Hospital’s rule differentiate the numerator
⎛ x⎞ 

32. lim ⎜ cos ⎟ is equal to and denominator separately.


n→∞ ⎝ n⎠
(A) e1 (B) e–1
(C) 1 (D) None of these CAUTION
Solution: (C) L’ Hospital’s rule cannot be applied in every problem.
⎛ x ⎞
⎛ x⎞
n lim n ⎜ cos -1⎟ 3 x + sin 2 x ⎛ ∞⎞
⎝ n ⎠ Consider the example, lim ⎜ form ⎟⎠ .
lim ⎜ cos ⎟ = e
n→∞

n→∞ ⎝
x →0 3x - sin 2 x ⎝ ∞
n⎠
⎛ x⎞ Here, if we apply L’ Hospital’s rule, we get
lim - n⋅2 sin 2 ⎜ ⎟
⎝ 2n⎠
= e 3 x + sin 2 x = 3 + 2 cos 2 x
n→∞

lim lim
x →∞ 3 x - sin 2 x x →∞ 3 - 2 cos 2 x
Limits 11.13

e x - e - x + 2 sin x ⎛0 ⎞
Now, both the numerator and denominator are undefined = lim ⎜⎝ form⎟⎠
x →0 24 x 0
because lim cos 2 x Does not exist.
x →∞
We can find the above limit as: e x + e - x + 2 cos x 4 1
= lim = = .
⎛ sin 2 x ⎞ x →0 24 24 6
3 + 2⎜
3 x + sin 2 x = ⎝ 2 x ⎟⎠ = 3 + 2(0)
lim lim 35. If f (2), g (x) be differentiable functions and f (1) =
x →∞ 3 x - sin 2 x x →∞ ⎛ sin 2 x ⎞ 3 - 2(0)
3 - 2⎜ ⎟ f (1) g ( x ) - f ( x ) g (1) - f (1) + g (1)
⎝ 2x ⎠ g (1) = 2 then lim
x →1 g( x) - f ( x)
sin 2 x is equal to
= 1, since lim =0
x →∞ 2 x (A) 0 (B) 1
(C) 2 (D) None of these
Solution: (C)
SOLVED EXAMPLES
f (1) g ( x ) - f ( x ) g (1) - f (1) + g (1) ⎛0 ⎞
lim ⎜⎝ form⎟⎠
x →1 g( x) - f ( x) 0
33. If a is a repeated root of ax2 + bx + c = 0, then
f (1) g’( x ) - f’( x ) g (1)
tan ( ax 2 + bx + c) = lim
lim is x →1 g’( x ) - f’( x )
x →a ( x - a )2
g’( x ) - f’( x )
(A) a (B) b (C) c (D) 0 = 2 lim
x →1 g’( x ) - f’( x )
Solution: (A)
= 2.
tan ( ax 2 + bx + c)
lim
x →a ( x - a )2 36. Let f (x) be a twice differentiable function and f ″ (0) = 5,
3 f ( x ) - 4 f (3 x ) + f ( 9 x )
⎛0 ⎞ then lim is equal to
⎜⎝ form as aa + ba + c = 0⎟⎠
2
0
x →0 x2
(A) 30 (B) 120
( 2ax + b) sec 2 ( ax 2 + bx + c) (C) 40 (D) None of these
= lim
x →a 2(x - a)
Solution: (B)
⎛0 ⎞
form as a being a repeated root of ax 2 + bx + c = 0, 3 f ( x ) - 4 f (3 x ) + f ( 9 x ) ⎛0 ⎞
⎜0 ⎟ lim
⎜ ⎟ ⎜⎝ form⎟⎠
⎝ 2aa + b = 0 ⎠ x →0 x2 0

2a sec 2 ( ax 2 + bx + c) + ( 2ax + b) 2 × 3 f’( x ) - 12 f’(3 x ) + 9 f’(9 x ) ⎛0 ⎞


= lim ⎜⎝ form⎟⎠
2 2 2 x →0 2x 0
2 sec ( ax + bx + c) tan( ax + bx + c)
= lim 3 f" ( x ) - 36 f" (3 x ) + 81 f" (9 x )
x →a 2 = lim
x →0 2
2a
= = a. 3 f" (0) - 36 f" (0) + 81 f" (0)
2 = = 24 f ″(0)
2
e x + e - x + 2 cos x - 4
34. lim is equal to = 24 ⋅ 5 = 120.
x →0 x4
(A) 0 (B) 1 (C)
1
(D) –
1 3- f ( x)
37. If f (9) = 9 and f ′ (9) = 1, then lim is equal
6 6 x →9 3- x
Solution: (C) to
-x
x
e +e + 2 cos x - 4 ⎛0 ⎞ (A) 0 (B) 1
lim 4 ⎜⎝ form⎟⎠ (C) –1 (D) None of these
x →0 x 0
e x - e - x - 2 sin x ⎛0 ⎞ Solution: (B)
= lim ⎜⎝ form⎟⎠
x →0 4 x3 0
3- f ( x) ⎛0 ⎞
lim ⎜⎝ form⎟⎠
e x + e - x - 2 cos x ⎛0 ⎞ x →9 3- x 0
= lim ⎜⎝ form⎟⎠
x →0 12 x 2 0
11.14 Chapter 11

1 Solution: (B)
0- ⋅ f ′( x )
= lim
2 f ( x) ⎛ ln cos x ⎞ ln [1 + (cos x - 1) ]
x →9 1 lim ⎜ ⎟ = xlim
0- x →0 ⎝ 4
1 + x - 1⎠
2 → 0 4
1 + x2 - 1
2 x
cos x - 1
[Using L’Hospital’s Rule] = 4 lim
x →0 x2

( )
x 3 ⎡
= lim ⋅ f’( x ) = × f ′ (9) = 1. x2 ⎤
⎢∵ ln [1 + (cos x - 1) ] ~ (cos x - 1) and
4
x →9 f ( x) 3 1 + x2 - 1 ~ ⎥
⎣ 4⎦
sin 2 x + a sin x ⎡
38. If lim be finite, then the value of a and x2/ 2 x2 ⎤
x →0 x 3 = –4 lim ⎢∵(1 - cos x ) ~ ⎥
the limit are given by
x →0 x2 ⎣ 2⎦
= –2
(A) – 2, 1 (B) – 2, –1
(C) 2, 1 (D) 2, –1 2x - x2
41. lim is equal to
Solution: (B)
x→2 x x - 22
sin 2 x + a sin x log 2 - 1 log 2 + 1
Let k = lim (A) (B)
x →0 x3 log 2 + 1 log 2 - 1
(C) 1 (D) –1
2 cos 2 x + a cos x
= lim Solution: (A)
x →0 3x 2
2x - x2 ⎛0 ⎞
[Using L’Hospital’s Rule] lim x
x → 2 x - 22
⎜⎝ form⎟⎠
0
We require 2 cos2x + a cos x = 0 for x = 0 as denomi-
nator is zero. 2 x log 2 - 2 x 4 log 2 - 4 log 2 - 1
= lim = = .
\ a = –2. x→2 x
x (1 + log x ) 4 (1 + log 2) log 2 + 1
2 cos 2 x - 2 cos x ⎛ 0⎞
Hence, k = lim ⎜⎝ ⎟⎠ sin x - (sin x )sin x
x →0 3x 2 0 42. lim equals
x→
p 1 - sin x + ln sin x
-4 sin 2 x + 2 sin x ⎛ 0⎞ 2
= lim ⎜⎝ ⎟⎠ (A) 1 (B) 2 (C) 3 (D) 4
x →0 6x 0
-8 cos 2 x + 2 cos x -8 + 2 Solution: (B)
= lim = = –1. Let sin x = h, then as x → p/2, h → 1
x →0 6 6
39. lim x x is equal to \ given limit
x →0
h - hh 1 - hh - hh ln h
(A) 0 (B) 1 = lim = lim
h → 1 1 - h + ln h h →1 -1 + 1 / h
(C) – 1 (D) None of these
(Using L’ Hospital Rule)
Solution: (B)
Let y = lim x x - h - 2h ln h - hh - 1 - hh (ln h) 2
h h

x →0 = lim
h →1 - 1 / h2
log x ⎛∞ ⎞
⇒ log y = lim x log x = lim ⎜⎝ form⎟⎠ -1 - 1
x →0 x →0 1 x ∞ = =2
-1
1/ x
= lim = – lim x = 0 p a-x
x → 0 -1/ x 2 x →0 43. The value of lim a 2 - x 2 cot is
x→a 2 a+ x
⇒ y = e0 = 1.
2a 2a
(A) (B) –
⎛ ln cos x ⎞ p p
40. lim ⎜ ⎟ is equal to
x →0 ⎝ 4
1 + x 2 - 1⎠ 4a 4a
(C) (D) –
p p
(A) 2 (B) –2 (C) 1 (D) –1
Limits 11.15

Solution: (C) -2 x
p a-x 2 a2 - x 2
lim a 2 - x 2 cot (0 × ∞ form) = lim
x→a 2 a+ x x→a p a-x p 2a
- sec 2 × ×
2 a + x 2 2( a + x ) a 2 - x 2
a2 - x 2 ⎛0 ⎞
= lim ⎜⎝ form⎟⎠ 4a
x→a p a-x 0 =
tan p
2 a+ x

EXERCISES

Single Option Correct Type

⎛ ⎡100 x ⎤ ⎡ 99 sin x ⎤⎞ 1 - cos 2( x - 1)


1. The value of lim ⎜ ⎢ ⎥ + ⎢ x ⎥⎟ , where [⋅]
x → 0 ⎝ ⎣ sin x ⎦
7. lim
x -1
⎣ ⎦⎠ x →1

represents greatest integer function, is (A) exists and it equals 2


(A) 199 (B) 198 (B) exists and it equals – 2
(C) 0 (D) None of these (C) Does not exist because (x – 1) → 0
(D) Does not exist because left hand limit is not equal
2. If f (x) = sin x, x ≠ np, to right hand limit
= 2, x = np
x5
where n ∈Z and 8. The value of lim is
g(x) = x2 + 1, x ≠ 2,
x →∞ 5x
(A) 1 (B) –1
= 3, x = 2. (C) 0 (D) None of these
then lim g [ f ( x ) ] is 1
x →0
9. lim (cos x + sin x ) x is equal to
(A) 1 (B) 0 x →0
(C) 3 (D) Does not exist
(A) e (B) e2 (C) e–1 (D) 1
⎛ ⎞
3. The value of lim ⎜ x + x+ x - x ⎟ is 2 2 - (cos x + sin x )3
x →∞ ⎝ ⎠ 10. The value of lim is
1 x→
p 1 - sin 2 x
(A) 1 4
2
(C) 0 (D) None of these 3 2 1
(A) (B) (C) (D) 2
2 3 2
⎡ x +1 p ⎤
4. The value of lim x ⎢ tan -1 - is
x →∞ ⎣ x + 2 4 ⎥⎦ ln(1 + 2h) - 2 ln(1 + h)
11. The value of lim is
1 1 h→ 0 h2
(A) (B) – (C) 1 (D) – 1 (A) 1 (B) – 1
2 2
(C) 0 (D) None of these

n→∞
( )
5. lim cos p n2 + n , n ∈ Z is equal to
⎛ 1 e1/ n e 2 / n
12. The value of lim ⎜ + + + ... +
e( n -1)/ n ⎞
is
(A) 0 (B) 1 n→∞ ⎝ n n n n ⎟⎠
(C) – 1 (D) None of these
(A) 1 (B) 0 (C) e– 1 (D) e + 1
nk sin 2 ( n !)
6. lim 0 < k < 1, is equal to x sin ( x - [ x ])
n →∞ n+2 13. lim , where [⋅] denotes the greatest
(A) ∞ (B) 1
x →1 x -1
(C) 0 (D) None of these integer function, is equal to
11.16 Chapter 11

(A) 1 (B) –1 (A) (–1)n (B) (–1)n – 1


(C) ∞ (D) Does not exist (C) 0 (D) Does not exist
2 sin x - sin 2 x y3
14. If f (x) = ∫ x3
dx , x ≠ 0, then lim f’( x ) is
x →0
·23. lim
x →1 x3 - y 2 - 1
as (x, y) → (1, 0) along the line y =
y →0
(A) 0 (B) ∞ (C) –1 (D) 1 x – 1 is given by
(A) 1 (B) ∞
⎡x⎤ (C) 0 (D) None of these
⎢⎣ 2 ⎥⎦
15. lim (where [⋅] denotes the greatest integer 1 - 2 + 3 - 4 + 5 - 6 + ... - 2n
x → p / 2 ln(sin x )
24. lim is equal to
n →∞
function) n2 + 1 + 4 n2 - 1
(A) Does not exist (B) equals 1 1 1
(C) equals 0 (D) equals –1 (A) (B) –
3 3
16. lim lim (1 + cos 2 m n !p x ) is equal to 1
m→∞ n→∞ (C) – (D) None of these
5
(A) 2 (B) 1
(C) 0 (D) None of these ⎡ x 4 sin (1/ x ) + x 2 ⎤
25. The value of lim ⎢ ⎥ is
x→-∞
⎣ 1 + | x |3 ⎦
⎡ sin([ x - 3]) ⎤
17. lim ⎢ ⎥ , where [⋅] represents greatest (A) 1 (B) –1 (C) 0 (D) ∞
x → 0 ⎣ [ x - 3] ⎦

integer function, is 2 x + 23 - x - 6
26. lim is equal to
(A) 0 (B) 1 x→2 2 - x / 2 - 21- x
(C) Does not exist (D) sin 1
(A) 8 (B) 4
18. The values of constants a and b so that (C) 2 (D) None of these
⎛ x2 + 1 ⎞
lim ⎜ - ax - b⎟ = 0 are 8 ⎛ x2 x4 x2 x2 ⎞
x →∞ ⎝ x + 1 ⎠ 27. lim 8 ⎜ 1 - cos - cos + cos cos ⎟ is equal
x →0 x ⎝ 2 4 2 4⎠
(A) a= 1, b = –1 (B) a = –1, b = 1 to
(C) a= 0, b = 0 (D) a = 2, b = –1 1 1 1 1
(A) (B) – (C) (D) –
⎛ 1 1 1 1 ⎞ 16 16 32 32
19. lim ⎜ + + + ... + is equal to
n→∞ ⎝ 1 ⋅ 2 2 ⋅ 3 3⋅ 4 n( n + 1) ⎟⎠ 28. lim [log n -1 ( n) ⋅ log n ( n + 1) ⋅ log n +1 ( n + 2)… log n k
k
-1 ( n )]
n→∞
(A) 1 (B) –1 is equal to
(C) 0 (D) None of these (A) ∞ (B) n
(log x ) 2 (C) k (D) None of these
20. lim , n > 0 is equal to
x →∞ xn ⎡ 1 1 1 1 ⎤
29. lim ⎢ + + + ... + is equal
(A) 1 (B) 0 (C) – 1 (D) ∞ n→∞ ⎣1 ⋅ 3 3 ⋅ 5 5 ⋅ 7 ( 2n + 1) ( 2n + 3) ⎥⎦
21. If the rth term, tr, of a series is given by to
n
1
r (A) 1 (B)
tr = 4
r + r2 + 1
, then lim
n→∞
∑ t r is
1
2
r =1 (C) – (D) None of these
1 2
nx
(A) 1 (B) ⎡11/ x + 21/ x + 31/ x + ... + n1/ x ⎤
2 30. The value of lim ⎢ ⎥ is
1 x →∞ n
(C) (D) None of these ⎣ ⎦
3
(A) n! (B) n (C) (n – 1)! (D) 0
22. lim( -1)[ x ] , where [x] denotes the greatest integer less
x→n 31. lim (1 + x) (1 + x2) (1 + x4) … (1 + x2n), | x | < 1, is
than or equal to x, is equal to n→∞
equal to
Limits 11.17

1 1 40. If x1 = 3 and xn + 1= 2 + x n , n≥ 1, then lim xn is


(A) (B) (C) 1 – x (D) x – 1 n→∞
x -1 x -1 equal to
xn (A) –1 (B) 2 (C) 5 (D) 3
32. lim = 0, (n integer), for
x →∞ ex 3x +1 - 5 x +1
41. The value of lim is
(A) no value of n x →∞ 3x - 5 x
(B) all values of n 1
(A) 5 (B)
(C) only negative values of n 5
(D) only positive values of n (C) –5 (D) None of these
n n -1 n-2 2
x +x +x + ... + x + x - n 1⎛
n -1 ⎞
33. The value of lim is 42. lim ⎜1 + e1/ n + e 2 / n + ... + e
x →1 x -1
n
⎟ is equal to
n→∞ n
⎝ ⎠
n ( n +1)
(A) (B) 0 (A) e (B) – e
2
(C) 1 (D) n (C) e– 1 (D) 1 – e

12 + 22 + 32 + ... + r 2 n x + sin x
34. If tr =
13 + 23 + 33 + ... + r 3
and Sn= ∑ ( -1)r ⋅ tr, then 43. lim
x →∞ x - cos x
=
r =1
lim Sn is given by (A) 0 (B) 1
n→∞
(C) – 1 (D) None of these
2 2 1 1
(A) (B) – (C) (D) – n Sn + 1 - Sn
3 3 3 3 44. If Sn= ∑ ai and lim an = a, then lim is
n→∞ n→∞ n
i =1
35. If lim
(1 + a3 ) + 8e1 x
= 2, then ∑i
x → 0 1 + (1 - b3 ) e1 x equal to i =1

(A) a = 1, b = (–3)1/3 (B) a = 1, b = 31/3 (A) 0 (B) a


(C) a = –1, b = –(3)1/3 (D) None of these (C) 2a (D) None of these
1 - cos 2q
36. If a = min {x2 + 4x + 5, x∈R} and b = lim 45. The value of lim ⎡ n2 - n3 + n⎤ is
, 3
n q →0 q2 n→∞ ⎢
⎣ ⎥⎦
then the value of ∑ ar ⋅ bn- r is (A)
1
(B) –
1
(C)
2
(D) –
2
r =0
n +1 3 3 3 3
2 -1
(A) (B) 2n + 1 – 1
4 ⋅ 2n 4
n5 + 2 - n2 + 1
3

n +1 46. The value of lim is


2 -1 n→∞ 5
n 4 + 2 - 2 n3 + 1
(C) (D) None of these
3 ⋅ 2n (A) 1 (B) 0 (C) –1 (D) ∞
1⋅ 2 + 2 ⋅ 3 + 3 ⋅ 4 + ... + n ( n + 1) (cos x - 1)(cos x - e x )
37. lim 3
is equal to 47. The integer n for which lim is
n→∞ n x→0 xn
(A) 1 (B) –1 a finite non-zero number, is
1 (A) 1 (B) 2 (C) 3 (D) 4
(C) (D) None of these
3
2 x +3 3 x +5 5 x
log (1 + x + x 2 ) + log (1 - x + x 2 ) 48. The value of lim is
38. lim is equal to x →∞ 3x - 2 + 3 2 x - 3
x →0 sec x - cos x 2
(A) (B) 3
(A) 1 (B) –1 (C) 0 (D) ∞ 3
ln x - 1 1
39. lim is equal to (C) (D) None of these
x →e| x-e| 3
1 1 x 3
z 2 - ( z - x)2
(A) (B) – 49. lim is equal to
( )
e e x→0 3 4
(C) e (D) Does not exist 8 xz - 4 x 2 + 3 8 xz
11.18 Chapter 11

(A)
z
(B)
1 43n - 2 - 9 n + 1
56. lim =
211/ 3 223/ 3 ⋅ z 82 n - 1 - 9 n - 1
n→ 0

(C) 221/3 z (D) None of these 1


(A) (B) 81
50. In a circle of radius r, an isosceles triangle ABC is 2
(C) Does not exist (D) None of these
inscribed with AB = AC. If the D ABC has perimeter P =
x - ai
57. If Ai = , i = 1, 2, …, n and if a1 < a2 < a3 … < an.
2 ⎡ 2 hr - h2 + 2 hr ⎤ and area A = h 2 hr - h2 , | x - ai |
⎣⎢ ⎦⎥
A
where h is the altitude from A to BC, then lim 3 is Then, lim ( A1 A2 ... An ), 1 ≤ m ≤ n
x → am
h→ 0 P +

equal to (A) is equal to (–1)m


1 (B) is equal to (–1)m +1
(A) 128 r (B)
128 r (C) is equal to (–1)m – 1
1
(C) (D) None of these (D) Does not exist
64 r
⎛ ⎡100 x ⎤ ⎡ 99 sin x ⎤⎞
⎛ 1 - cos{2( x - 2)} ⎞ 58. The value of lim ⎜ ⎢ ⎥ + ⎢ x ⎥⎟ , where [⋅]
x → 0 ⎝ ⎣ sin x ⎦
51. lim ⎜ ⎣ ⎦⎠
x→2⎝

x-2 ⎠ represents greatest integer function, is
1 (A) 199 (B) 198
(A) equals (B) Does not exist
2 (C) 0 (D) None of these
1 x5
(C) equals (D) equals - 2 59. The value of lim x is
x →∞ 5
2
(A) 1 (B) –1
⎛ x x x x⎞ (C) 0 (D) None of these
52. lim ⎜ cos cos cos ... cos n ⎟ =
n→∞ ⎝ 2 4 8 2 ⎠ 60. If the rth term, tr, of a series is given by
x sin x r n
(A)
sin x
(B)
x
tr= 4
r + r2 + 1
, then lim
n→∞
∑ t r is
r =1
(C) 0 (D) None of these 1
(A) 1 (B)
53. The value of 2
1
(C) (D) None of these
1 ⎡ ⎛ n ⎞ ⎛ n -1 ⎞ ⎛n-2 ⎞ ⎤ 3
lim ⎢1
n→∞ n 4 ⎢ ⎝
⎜ ∑ k ⎟ + 2 ⎜ ∑ k ⎟ + 3 ⎜ ∑ k ⎟ + ... + n ⋅ 1⎥
⎥⎦
⎡11/x + 21/x + 31/x + ... + n1/x ⎤
nx

⎣ k =1 ⎠ ⎝ k =1 ⎠ ⎝ k =1 ⎠ 61. The value of lim ⎢


x →∞
⎥ is
⎣ n ⎦
will be
1 1 1 1 (A) n! (B) n (C) (n – 1)! (D) 0
(A) (B) (C) (D)
24 12 6 3 62. lim (1 + x )(1 + x 2 )(1 + x 4 )… (1 + x 2 n ), | x | < 1, is equal to
n→∞
54. If [x] denotes the integral part of x, then 1 1
(A) (B)
1 ⎛ n ⎞ x -1 1- x
lim 3 ⎜ ∑ [k 2 x ]⎟ =
n→∞ n ⎝ ⎠ (C) 1 – x (D) x – 1
k =1
x x x xn
(A) 0 (B) (C) (D) 63. lim = 0, (n integer), for
2 3 6 x →∞ ex
(A) no value of n
⎛ 1 q 1 q 1 q⎞ (B) all values of n
55. lim ⎜ tan q + tan + 2 tan 2 + … + n tan n ⎟ =
n→∞ ⎝ 2 2 2 2 2 2 ⎠ (C) only negative values of n
1 1 (D) only positive values of n
(A) (B) - 2 cot 2q 1 - cos 2q
q q 64. If a = min {x2 + 4x + 5, x ∈R} and b = lim
(C) 2 cot 2q (D) None of these
n q →0 q2
then the value of ∑ a ⋅b
r n-r
is
r=0
Limits 11.19

2n + 1 - 1 1 - cos x
(A) (B) 2n + 1 – 1 72. lim =
4 ⋅ 2n x→ 0 x
2n + 1 - 1 1 1
(C) (D) None of these (A) (B) –
3 ⋅ 2n 2 2
log (1 + x + x 2 ) + log (1 - x + x 2 ) (C) Does not exist (D) None of these
65. lim is equal to
x →0 sec x - cos x x + 7 - 3 2x - 3
73. lim =
(A) 1 (B) –1 (C) 0 (D) ∞ x→ 2 3 x + 6 - 2 3 3x - 5
4
n5 + 2 - 3 n 2 + 1 17 34
66. The value of lim is (A) (B)
n→∞ 5 23 23
n4 + 2 - 2 n3 + 1
(C) 1 (D) None of these
(A) 1 (B) 0 (C) –1 (D) ∞
( 2m + x )1/m - ( 2n + x )1/n
2 2 74. lim is equal to
x 3
z - ( z - x) x→ 0 x
67. lim is equal to
( 8xz - 4 x ) 1 1 1 1
x →0 4
3 2
+ 8 xz
3 (A) m
- n (B) m
+ n
m2 n2 m2 n2
z 1 1 1
(A) 11/3
(B) 23/3 (C) - (D) None of these
2 2 ⋅z m2m -1 n2n -1
21/3
(C) 2 z (D) None of these
(cos q ) x - (sin q ) x - cos 2q
75. lim =
68. In a circle of radius r, an isosceles triangle ABC is x→ 4 x-4
inscribed with AB= AC. If the DABC has perimeter P = (A) cos4 q lncos q – sin4 q ln sin q
2 ⎡ 2hr - h2 + 2hr ⎤ and area A = h 2hr - h2 , (B) cos4 q lncos q + sin4 q ln sin q
⎣⎢ ⎦⎥ (C) cos4 q ln sin q – sin4 q lncos q
A
whereh is the altitude from A to BC, then lim 3 is (D) None of these
h→ 0 P +

equal to 1/ x
1 ⎛ x - 1 + cos x ⎞
(A) 128r (B) 76. lim ⎜ ⎟⎠ =
128r x→ 0 ⎝ x
1
(C) (D) None of these (A) e1/2 (B) e–1/2
64r
(C) e1/4 (D) None of these
⎛ p⎞
cos ⎜ x + ⎟ ⎡ ⎤
x
⎝ 6⎠ e
69. lim = 77. lim ⎢ ⎥ =
x →p /3 (1 - 2 cos x ) 2/3 ⎣ ( )
x →∞ ⎢ 1 + 1/ x x ⎥

(A) 1 (B) –1
(A) e (B) e–1 (C) e1/2 (D) e–1/2
(C) 0 (D) None of these
ln ( 2 - cos 2 x ) ⎡ a sin x ⎤ ⎡ b tan x ⎤
78. lim ⎢ + , where a, b are integers and
70. lim 2
is equal to x→ 0 ⎣ x ⎥⎦ ⎢⎣ x ⎥⎦
x →0ln (sin 3 x + 1)
[ ] denotes integral part, is equal to
2 2
(A) (B) – (A) a+ b (B) a + b – 1
9 9 (C) a– b (D) a – b – 1
(C) 0 (D) None of these
[ x ] + [2 x ] + [3 x ] + ... + [nx ]
1 - cos (cx 2 + bx + a) 79. lim =
71. lim , where a is a root of n→∞ 1 + 2 + 3 + ... + n
x → 1/a (1 - xa ) 2
2 (A) x (B) 2x
ax  + bx+ c = 0, is equal to
(C) 0 (D) None of these
b 2 - 4 ac b 2 - 4 ac
(A)
2a 2
(B)
a2 n→∞
( )
80. lim n2 x1/n - x1/n + 1 , x > 0 is equal to

4 ac - b 2 (A) 0 (B) ex
(C) (D) None of these
2a 2 (C) lnx (D) None of these
11.20 Chapter 11

⎡ f ( x) ⎤
1/ x
⎡ f ( x) ⎤
1/ x (A) 0 (B) 1
3
81. If lim ⎢1 + x + = e , then lim ⎢1 + =
x ⎥⎦ x ⎥⎦
(C) –1 (D) Does not exist
x→ 0 ⎣ x→ 0 ⎣
_ n ≥ 2, then
91. If a1 = 1 and an= n (1 + an – 1), ∨
(A) e (B) e2
(C) e3 (D) None of these ⎛ 1⎞ ⎛ 1⎞ ⎛ 1⎞
lim ⎜1 + ⎟ ⎜1 + ⎟ ... ⎜1 + ⎟ =
n→∞ ⎝ a1 ⎠ ⎝ a2 ⎠ ⎝ an ⎠
x x
82. If y = x + , then lim is equal to (A) 0 (B) e
x x →∞ y
x+ (C) e2 (D) Does not exist
x
x+ ⎡ 1 ⎞⎤
n
... ∞ - n2 ⎛ 1⎞ ⎛ 1⎞ ⎛
(A) 1 (B) –1
92. lim n
n→∞
⎢( n + 1) ⎜⎝ n + ⎟⎠ ⎜⎝ n + 2 ⎟⎠ ... ⎜⎝ n + n -1 ⎟⎠ ⎥
⎣ 2 2 2 ⎦
(C) 0 (D) None of these
(A) e (B) e2
cos x - (cos x )cos x (C) e4 (D) None of these
83. lim =
x → 0 1 - cos x + ln (cos x )
x y - yx 1- k
(A) 0 (B) 1 93. If lim = , then k =
x→ y x -yx y 1+ k
(C) 2 (D) None of these
(A) log y (B) ey
(tan x ) tan x - tan x (C) y (D) None of these
84. The value of lim is
x → p /4 ln (tan x ) - tan x + 1
⎛ 3 1⎞
n r -r +
(A) –2 (B) 1 ⎜ r ⎟ is equal to
(C) 0 (D) None of these 94. lim ∑ cot -1 ⎜ ⎟
n→∞ 2
r =1 ⎜ ⎟
⎝ ⎠
( )
2 2 2 cos 2 x
85. lim 11/cos x
+ 21/cos x
+ ... + n1/cos x
= (A) 0 (B) p
x → p /2
n ( n + 1) p
(A) n (B) (C) (D) None of these
2 2
(C) n! (D) None of these ⎛ ⎡ n sin x ⎤ ⎡ n tan x ⎤⎞
95. The value of lim ⎜ ⎢ + , where [·]
n
⎛ 3⎞ n→∞ ⎝ ⎣ x ⎥⎦ ⎢⎣ x ⎥⎦⎟⎠
86. lim
n→∞
∑ cot -1 ⎜⎝ r 2 + 4 ⎟⎠ =
denotes the greatest integer function, is
r =1
(A) 0 (B) tan–12 (A) n (B) 2n + 1
p (C) 2n – 1 (D) None of these
(C) (D) None of these
4 ⎡ x2 ⎤
87. The value of lim [sin x + cos x], where [·] denotes 96. lim ⎢ ⎥ , where [·] denotes the greatest
x → 0 sin x tan x
x→ 5p / 4 ⎣ ⎦
the greatest integer function, is integer function, is
(A) 2 (B) –2 (C) 1 (D) –1 (A) 0 (B) 1
(C) 2 (D) Does not exist
88. lim lim
m→∞ n→∞
cos 2 (1 - cos 2 (1 - cos 2 (1 ... cos 2 q ))
⎛ 1 + n 1n + 2n + n 2n + 3n + n 3n + 4 n + … + n ( m - 1) n + m n ⎞ 97. lim =
q→ 0 ⎛ p ( q + 4 - 2) ⎞
⎜ ⎟ sin ⎜
⎜⎝ m2 ⎟⎠ q ⎟
⎝ ⎠
1
(A) 0 (B) 1 (C) –1 (D) (A) 1 (B) 0 (C) 2 (D) – 2
2
⎡ n 1⎤
89. The value of lim ⎢ ∑ r ⎥ , where [·] denotes the tan x - sin{tan -1 (tan x )}
n→∞ ⎢
⎣ r = 1 2 ⎥⎦ 98. Let f (x) = , then lim f ( x ) =
greatest integer, is tan x + cos 2 (tan x ) x→
p
2
1 (A) 1
(A) 0 (B) 1 (C) –1 (D)
2 (B) –1
90. The value of lim | x | [cos x ] , where [·] denotes the (C) 0
x →∞
greatest integer, is (D) Does not exist
Limits 11.21

⎧ ⎡ 1/2 1/2 -1 -1 103. If a and b are the roots of the quadratic equation

⎪ ⎢⎛ a + x ⎞ 2( ax )1/4
99. lim ⎨ ⎜ 1/2 1/4 ⎟ - 3/4 1/4 1/2 1/2 1/4 3/4 ⎥
x → a ⎢⎝ a - x ⎠ x -a x +a x -a ⎥ 1 - cos( cx 2 + bx + a)
⎪⎣ ⎦ ax2 + bx+ c = 0, then lim =
⎩ x→
1 2(1 - a x ) 2
a
⎫⎪4

-2log ⎬ =a
c ⎛ 1 1⎞ c ⎛ 1 1⎞
⎭⎪ (A) - (B) -
2a ⎜⎝ a b ⎟⎠ 2b ⎜⎝ a b ⎟⎠
(A) a3/4 (B) a
(C) a2 (D) None of these c ⎛ 1 1⎞
(C) - (D) None of these
(log (1 + x ) - log 2) (3.4 x -1
- 3x) ab ⎜⎝ a b ⎟⎠
100. lim =
x →1 {(7 + x )1/3 - (1 + 3 x )1/2 } sin p x 104. Given a real valued function f such that
9 4 3 4 ⎧ tan 2 {x}
(A) log (B) log
p e p e ⎪ 2 , x>0
⎪ x - [ x]
2
9 2 ⎨
(C) log (D) None of these f (x) = ⎪ 1 , x=0
p e
⎪ {x} cot {x} , x < 0
(1 - x ) (1 - x 2 ) ... (1 - x 2 n ) ⎩
101. lim = 2
x →1 [(1 - x ) (1 - x 2 ) ... (1 - x n )]2 ⎛ ⎞
–1 ⎜
f ( x )⎟ is
( 2n)! The value of cot ⎝ xlim
→0 ⎠
(A) n! (B)
n! (A) 0 (B) 1
( 2n)!
(C) (D) None of these (C) –1 (D) None of these
( n !) 2 x a sin b x
105. If lim , a, b, c∈ R – {0} exists and has
k
kp x →0sin ( x c )
102. If ∑ cos a r = -1
for any k ≥ 1
r =1 2 non-zero value, then
k
(1 + x 2 )1/3 - (1 - 2 x )1/4 (A) a, b, c are in A.P.
and q = ∑ (a r ) r , then lim is
r =1
x →q x + x2 (B) a, c, b are in A.P.
1 1 (C) a, c, b are in G.P.
(A) 1 (B) –1 (C) (D) – (D) None of these
2 2

More than One Option Correct Type


3x
106. lim lim (1 + cos 2 m n !p x ) is equal to ⎛ a1/x + b1/x + c1/x ⎞
m →∞ n →∞ 108. lim ⎜ ⎟ =
x →∞ ⎝ 3 ⎠
(A) 2 (B) 1
(C) 0 (D) None of these (A) (a + b + c) (B) elog (a + b + c)
(1 + a3 ) + 8e1/x (C) abc (D) elog (abc)
107. If lim = 2, then
x→0 1 + (1 - b3 ) e1/x axe x - b log(1 + x ) + cxe - x
109. If lim = 2, then
x→0 x 2 sin x
(A) a = –1 (B) a = 1 (A) a = 3 (B) b = 12
(C) b = (–3)1/3 (D) 31/3 (C) c = 9 (C) a = –3

Passage Based Questions


Passage 1 However, if lim g ( x ) = 0 = lim f ( x ) , we cannot say
x→a x→a
We know that if lim f ( x ) = l and lim g ( x ) = m(≠ 0), then f ( x)
x→a x→a anything definite about the existence of lim . Though
x → a g( x)
lim f ( x ) in some cases this limit exists. Any expression of the type
f ( x) x→a
lim = 0 ∞
x→a g( x) lim g ( x ) or is termed as an indeterminate form. Many other
x→a 0 ∞
11.22 Chapter 11

expressions like ∞ – ∞, 1∞, ∞0, 00, 0 × ∞ which can be 112. lim | x | sin x equals
x →0
0 ∞
reduced to the form or are also called indeterminate (A) 0 (B) 1
forms. 0 ∞ (C) –1 (D) None of these
f ( x) 0 ∞
If is indeterminate at x = a of the type or , 113. If a and b be the roots of ax2 + bx+ c = 0, then
g( x) 0 ∞
then lim (1 + ax 2 + bx + c)1/( x - a ) is
x →a
f ( x) f ′( x )
lim = lim , (A) log |a(a – b )| (B) ea(a – b)
x → a g( x) x → a g ′( x )

wheref ′ is derivative of f. (C) ea(b – a) (D) None of these


f ′( x ) Passage 3
If , too, is indeterminate at x = a of the type
g ′( x ) Let f, g and h be real valued functions defined on an interval
0 ∞ f ′( x ) f ′′( x )
or , then lim = lim I ⊆ R except possibly for some point c such that
0 ∞ x → a g ′( x ) x → a g ′′( x )
lim f ( x ) = l = lim h( x )
This can be continued till we finally arrive at a deter- x→c x→c
minate result. and, f (x) ≤ g(x) ≤ h(x), ∀ x ∈ I.
sin 2 x + a sin x
110. If lim be finite, then the value of a Then, lim g ( x ) = l.
x→0 x3 x→c
and the limit are given by 1⋅ 3 ⋅ 5… ( 2n - 1)
(A) –2, 1 (B) –2, –1 (C) 2, 1 (D) 2, –1 114. lim =
n→δ +∞ 2 ⋅ 4 ⋅ 6… 2n

p a-x (A) 1 (B) –1


111. The value of lim a 2 - x 2 cot is (C) 0 (D) None of these
x→0 2 a+ x
2a 2a 4a 4a {x} + {2 x} + {3 x} + ... + {nx}
(A) (B) - (C) (D) - 115. lim ,
p p p p n →∞ n2
Passage 2 where {x} = x – [x] denotes the fractional part of x, is
For a function f, let lim f ( x ) ≠ 1 but f (x) is (A) 1 (B) 0
x→a 1
(C) (D) None of these
lim
x→a
{ f ( x)} g ( x ) , 2
g(x) ⎛ [12 x x ] + [22 x x ] + ... + [n2 x x ] ⎞
we write { f (x)}g(x) = e log { f ( x )}
e
116. lim ⎜ lim ⎟ , where [·]
lim g ( x ) log f ( x) x → 0 ⎝ n →∞
+
n3 ⎠
⇒ lim { f ( x )}g ( x ) = e x→a e

x→a denotes the greatest integer function, is equal to


In case, lim f ( x ) = 1 and lim g ( x ) = ∞, then 1 1
x→a x→a (A) – (B)
g( x) 3 3
lim { f ( x )} = lim (1 + f ( x ) - 1) g ( x )
x→a x→a (C) 0 (D) None of these
lim ( f ( x ) - 1) g ( x )
= e x→a

Match the Column Type


117. n ⎛ r 3 - 1⎞ 1
Column-I Column-II
(III) lim
n →∞
∏ ⎜⎝ r 3 + 1⎟⎠ (C)
3
r=3
1
(I) lim ⎡ 3 n2 - n3 + n⎤
n
(A) ⎛ x⎞
n→∞ ⎢
⎣ ⎥⎦ 9 (IV) lim ⎜ cos ⎟ (D) 1
n →∞ ⎝ n⎠
3 2
x - 2 x +1
3
6
(II) lim (B)
x →1 ( x - 1) 2 7
Limits 11.23

118.
⎡ x +1 p⎤
(III) lim x ⎢ tan -1 - (C) 0
x + 2 4 ⎥⎦
Column-I Column-II
x →∞ ⎣
sin x - (sin x )sin x
(I) lim (A) 2 nk sin 2 ( n !) 5
x→
p 1 - sin x + ln sin x (IV) lim ,0 < k <1 (D)
2 n→∞ n+2 4
⎧ 7 29 133 1
(II) lim ⎨ + 2 + 3 + … (B) –
n→∞ ⎩10 10 10 2
5n + 2n ⎫⎪
+ ⎬
10 n ⎭⎪

Assertion-Reason Type
Instructions: In the following questions an Assertion (A) is e - (1 + x )1/ x e
given followed by a Reason (R). Mark your responses from 122. Assertion: lim =
x →0 x 2
the following options:
(A) Assertion(A) is True and Reason(R) is ln (1 + x ) - x
Reason: lim =0
True; Reason(R) is a correct explanation for x→0 x
Assertion(A) ln (1 + x ) - x 1
(B) Assertion(A) is True, Reason(R) is True; and lim 2
=–
x→0 x 2
Reason(R) is not a correct explanation for 1
Assertion(A)
(C) Assertion(A) is True, Reason(R) is False
( )
123. Assertion: lim [ f ( x )] + x 2 { f ( x )} = e, where f (x) =
x→0
tan x
(D) Assertion(A) is False, Reason(R) is True and [·], { } denote integral and fractional parts,
x
12 + 22 + 32 + ... + r 2 respectively
119. Assertion: If tr= and
13 + 23 + 33 + ... + r 3 ⎡ tan x ⎤ 2
n ⎢⎣ x ⎥⎦ + x - 1
2 =3
Sn= ∑ ( -1) r . t r , then lim Sn = Reason: lim
x→0 ⎧ tan x ⎫
n →∞ 3
r =1 ⎨ ⎬
r ( r + 1) ( 2r + 1) ⎩ x ⎭
Reason: 12 + 22 + 32 + … + r2 =
6 1 x 1 x 1 x
2 124. Assertion: lim tan + 2 tan 2 + ... + n tan n
⎛ r ( r + 1 ) ⎞ n →∞ 2 2 2 2 2 2
and 13 + 23 + 33 + … + r3 = ⎜ ⎟
⎝ 2 ⎠ 1
= –cot x +
x
120. Assertion: If x1 = 3 and xn + 1 = 2 + xn , n ≥ 1, then
1 x 1 x
lim xn = 2 Reason: cot x + tan = cot
n →∞ 2 2 2 2
Reason: A monotonically decreasing sequence which cot q tan -1 ( m tan q ) - m cos 2 (q/2)
is bounded below is convergent 125. Assertion: lim
q →0 sin 2 (q/2)
1
n →∞ n
(
121. Assertion: lim 1 + e1/n + e 2/n + ... + e( n -1)/n = e – 1) =m–
4 3
m
3
Reason: 1 + r + r2 + … + rn – 1 x - tan x 1
Reason: lim =
⎧1 - r n x→0 x 3 3
⎪ if r <1
⎪ 1- r
= ⎨ .
⎪r -1
n

⎪⎩ r - 1 if r >1
11.24 Chapter 11

Previous Year’s Questions

1 - cos 2 x ⎛ a b⎞
2x
126. lim is [2002] 133. If lim ⎜1 + + 2 ⎟ = e 2 , then the values of a and b,
x →∞ 2x x →∞ ⎝ x x ⎠
(A) l (B) − 1 are [2004]
(C) Zero (D) Does not exist
(A) a ∈ R, b ∈ R (B) a = 1, b ∈ R

⎛ x 2 + 5x + 3⎞
x
(C) a ∈ R, b = R (D) a = 1 and b = 2
127. lim ⎜ 2 ⎟ is equal to [2002]
x →∞ ⎝ x + x + 2 ⎠ 134. Let a and b be the distinct roots of ax2 + bx + c = 0,
(A) e4 (B) e2 1 - cos( ax 2 + bx + c)
then lim is equal to [2005]
(C) e3 (D) e x →a ( x - a )2
2
⎛ x - 3⎞ a2
128. For x ∈ R, lim ⎜ ⎟ is equal to [2002] (A) (a - b ) 2 (B) 0
x →∞ ⎝ x + 2 ⎠ 2
(A) e (B) e−1
a2 1
(B) e− 5
(D) e5 (C) - (a - b ) 2 (D) (a - b ) 2
2 2
xf ( 2) - 2 f ( x ) 135. Let f : R → R be a positive increasing function such
129. Let f (2) = 4 and f ′ (2) = 4. Then lim is
x→2 x-2 f (3 x ) f (2 x)
given by [2002] that lim = 1 . Then, lim = [2010]
x →∞ f ( x ) x →∞ f ( x )
(A) 2 (B) − 2
(C) − 4 (D) 3 2 3
(A) (B)
3 2
⎡ ⎛ x⎞ ⎤
⎢1 - tan ⎜⎝ 2 ⎟⎠ ⎥ [1 - sin x ] (C) 3 (D) 1
130. lim ⎣ ⎦ is [2003] ⎛ 1 - cos{2( x - 2)} ⎞
x →p / 2 ⎡ ⎛ ⎞⎤
x 136. Limit of ⎜ ⎟ as x tends to 2 [2011]
⎢1 + tan ⎜⎝ 2 ⎟⎠ ⎥ [p - 2xx ]
3
⎝ x-2 ⎠
⎣ ⎦
1 (A) equals 2 (B) equals – 2
(A) (B) 0
8
1
1 (C) equals (D) does not exist
(C) (D) ∞ 2
32
131. If lim
log(3 + x ) - log(3 - x )
= k , the value of k is 137. The value of lim
(1 - cos 2 x ) (3 + cos x ) is equal to
x →0 x x →0 x tan 4 x
[2003] [2013]
1 1
(A) 0 (B) - (A) (B) 1
3 2
2 2 1
(C) (D) - (C) 2 (D) -
3 3 4
138. Let f (x) be a forth degree polynomial having extreme
132. Let f (a) = g(a) = k and their nth derivatives f n(a),
gn(a) exist and are not equal for some n. Further if ⎡ f ( x) ⎤
values at x = 1 and x = 2. If lim ⎢1 + 2 ⎥ = 3 , then
f ( a) g ( x ) - f ( a) - g ( a) f ( x ) + g ( a) x →0 ⎣ x ⎦
lim = 4 , then the f (2) is equal to [2015]
x→a g( x) - f ( x)
value of k is [2003] (A) -4 (B) 0
(C) 4 (D) -8
(A) 4 (B) 2
(C) 1 (D) 0
Limits 11.25

(1 - cos 2 x ) (3 + cos x ) 1
139. The value of lim is equal to 140. Let p = lim (1 + tan 2 x ) 2 x then log p is equal to
x →0 x tan 4 x x →0 +
[2015] [2016]
(A) 3 (B) 2 1
(A) (B) 2
1 4
(C) (D) 4 1
2 (C) 1 (D)
2

ANSWER KEYS

Single Option Correct Type


1. (B) 2. (A) 3. (A) 4. (B) 5. (A) 6. (C) 7. (D) 8. (C) 9. (A) 10. (A)
11. (B) 12. (C) 13. (D) 14. (D) 15. (C) 16. (a, b) 17. (C) 18. (A) 19. (A) 20. (B)
21. (B) 22. (D) 23. (C) 24. (B) 25. (B) 26. (A) 27. (C) 28. (C) 29. (B) 30. (A)
31. (B) 32. (B) 33. (A) 34. (B) 35. (A) 36. (B) 37. (C) 38. (A) 39. (D) 40. (B)
41. (A) 42. (C) 43. (B) 44. (A) 45. (A) 46. (B) 47. (C) 48. (A) 49. (B) 50. (B)
51. (B) 52. (B) 53. (A) 54. (C) 55. (B) 56. (C) 57. (D) 58. (B) 59. (C) 60. (B)
61. (A) 62. (B) 63. (B) 64. (B) 65. (A) 66. (B) 67. (B) 68. (B) 69. (C) 70. (A)
71. (A) 72. (C) 73. (B) 74. (C) 75. (A) 76. (B) 77. (C) 78. (B) 79. (A) 80. (C)
81. (B) 82. (A) 83. (C) 84. (A) 85. (A) 86. (B) 87. (B) 88. (D) 89. (A) 90. (B)
91. (B) 92. (B) 93. (A) 94. (C) 95. (C) 96. (A) 97. (C) 98. (A) 99. (C) 100. (A)
101. (C) 102. (C) 103. (A) 104. (D) 105. (D)

More than One Option Correct Type


106. (A) and (B) 107. (B) and (C) 108. (C) and (D) 109. (A), (B) and (C)

Passage Based Questions


110. (B) 111. (C) 112. (B) 113. (B) 114. (C) 115. (B) 116. (B)

Match the Column Type


117. (I) → (C); (II) → (A); (III) → (B); (IV) → (D)
118. (I) → (A); (II) → (D); (III) → (B); (IV) → (C)

Assertion-Reason Type
119. (D) 120. (A) 121. (A) 122. (A) 123. (D) 124. (A) 125. (C)

Previous Year’s Questions


126. (D) 127. (A) 128. (C) 129. (C) 130. (C) 131. (C) 132. (A) 133. (B) 134. (A) 135. (D)
136. (D) 137. (C) 138. (A) 139. (B) 140. (D)
11.26 Chapter 11

HINTS AND SOLUTIONS

Single Option Correct Type


sin x x ⎡ 1 ⎤
→ 1– and lim → 1+ ⎛ 1⎞ 2
5. lim cos ⎡p n2 + n ⎤ = lim cos ⎢ np ⎜1 + ⎟ ⎥
1. We know that lim
x→0 x → sin x
x 0
n→∞ ⎢⎣ ⎥⎦ n→∞ ⎢ ⎝ n⎠ ⎥
⎡ x ⎤ ⎡ sin x ⎤ ⎣ ⎦
\ lim ⎢100 ⎥ + lim 99 = 100 + 98 = 198.
x→0⎣ sin x ⎦ x → 0 ⎢⎣ x ⎥⎦ ⎡ ⎛ 1 1 ⎞⎤
= lim cos ⎢ np ⎜1 + - 2 + ...⎟ ⎥
The correct option is (B) n→∞ ⎣ ⎝ 2 n 8n ⎠⎦
2. g [ f (x)] = [ f (x)]2 + 1, f (x) ≠ 2 ⎛ p p ⎞
= lim cos ⎜ np + - + ...⎟
3, f (x) = 2 n→∞ ⎝ 2 8n ⎠
\ g [ f (x)] = sin2x + 1, x ≠ np ⎛ p ⎞
3, x = np = – lim sin ⎜ np - + ...⎟
n→∞ ⎝ 8n ⎠
RHL = lim g [ f (0 + h)] = lim(sin 2 h + 1) = 1.
p
= – lim ( -1) n -1 sin ⎛⎜ - ...⎞⎟
h→ 0 h→ 0

LHL = lim g [ f (0 - h)] = lim (sin 2 h + 1) = 1. n→∞ ⎝ 8n ⎠


h→ 0 h→ 0
⎛ p ⎞
\ lim g [ f ( x )] = 1 =0 ⎜⎝∵ - ... → 0 as n → ∞⎟⎠
x→0 8n
The correct option is (A) The correct option is (A)
⎡ ⎤ nk sin 2 ( n!) nk sin 2 ( n!)
3. lim ⎢ x + x + x - x ⎥ 6. lim = lim
x→∞ ⎣ ⎦ n→∞ n+2 n→∞ ⎛ 2⎞
n ⎜1 + ⎟
⎝ n⎠
x+ x+ x -x
= lim
x→∞ sin 2 ( n!)
x+ x+ x + x = lim
n → ∞ 1- k ⎛ 2⎞
n ⎜1 + ⎟
x+ x ⎝ n⎠
= lim
x→∞ a finite quantity
x+ x+ x + x =

1/ 2
⎛ 1 ⎞ [∵ sin2 (n !) always lies between 0 and 1. Also,
x ⎜1 + ⎟
⎝ x⎠ since 1 – k > 0, \ n1 – k→ ∞ as n → ∞]
= lim
x→∞ ⎡⎛ 1 1 ⎞
1/ 2 ⎤ = 0.
x ⎢⎜1 + 1+ ⎟ + 1⎥
⎢⎝ x x⎠ ⎥ The correct option is (C)
⎣ ⎦
1 1 1 - cos 2( x - 1) 2 sin 2 ( x - 1)
= = 7. lim = lim
1+1 2 x →1 x -1 x →1 x -1
The correct option is (A) 2 sin( x - 1)
= lim
⎡ x +1 p ⎤ x →1 ( x - 1)
4. lim x ⎢ tan -1 -
x →∞ ⎣ x + 2 4 ⎥⎦ 2 sin( x - 1) 2 sin(1 - h - 1)
⎛ x +1 ⎞ LHL = lim = lim -
-1 ( x - 1) (1 - h - 1)
⎡ -1 x + 1 -1 ⎤ -1 ⎜ x + 2 ⎟ x →1- h→ 0
= lim x ⎢ tan - tan 1⎥ = lim x tan ⎜ ⎟
x →∞ ⎣ x+2 ⎦ x →∞ x + 1
⎜1+ ⎟ 2 - sin h sin h
⎝ x + 2⎠ = lim = – 2 lim
h→ 0 -h h→ 0 h
⎛ 1 ⎞
tan -1 ⎜
-1 ⎞
-1 ⎛ ⎝ 2 x + 3 ⎟⎠ 1 =– 2⋅1=– 2
= lim x tan ⎜ = – lim ⋅
x →∞ ⎝ 2 x + 3 ⎟⎠ x →∞ ⎛ 1 ⎞ ⎛ 3⎞
⎜⎝ ⎟ ⎜⎝ 2 + ⎟⎠ 2 sin( x - 1) 2 sin(1 + h - 1)
2 x + 3⎠ x RHL = lim = lim
1 1 x →1+ ( x - 1) h→ 0 (1 + h - 1)
= –1 × =–
2 2 2 sin h sin h
= lim = 2 lim = 2⋅1= 2
The correct option is (B) h→0 h h→0 h
Limits 11.27

Since LHL ≠ RHL, ⎛ h2 ⎞


ln ⎜1 + ⎟
1 - cos 2( x - 1) ⎝ 1 + 2h ⎠
\ lim does not exist. = – lim
h→ 0 ⎛ h2 ⎞
x →1 x -1
⎜ 1 + 2h ⎟ (1 + 2h)
The correct option is (D) ⎝ ⎠

x5 x5 x5 ⎛ h2 ⎞
8. lim = lim = lim , ln ⎜1 + ⎟
⎝ 1 + 2h ⎠
x →∞ 5 x x →∞ e x log 5 x →∞ e kx 1
= – lim ⋅ = –1
where k = log 5 h→ 0 ⎛ h ⎞ 2 1 + 2h
⎜ 1 + 2h ⎟
x5 ⎝ ⎠
= lim ⎡ log(1 + x ) ⎤
x →∞ ⎛ k 2 x 2 k 3 x 3 k 4 x 4 x 5k 5 k 6 x 6 ⎞
⎢ Using xlim = 1⎥
⎜1 + kx + 2! + 3! + 4! + 5! + 6! + ...⎟ ⎣ →0 x ⎦
⎝ ⎠
The correct option is (B)
1
= lim ⎡ 1 e1 n e 2 n e( n -1) n ⎤
x →∞ ⎡⎛ 1 1 k2 1 k3 1 k4 1⎞ 12. lim ⎢ + + + ... + ⎥
⎢⎜ 5 + k ⋅ 4 + ⋅ + ⋅ + ⋅ n→∞ ⎢ n
⎣ n n n ⎥⎦
⎢⎣⎝ x x 2! x 3 3! x 2 4! x ⎟⎠
⎡1 + e1 n + (e1 n ) 2 + ... + (e1 n ) n -1 ⎤
k5 ⎛ k6 ⎞⎤ = lim ⎢ ⎥
+ + ⎜ x + ...⎟ ⎥ n→∞ ⎢ n ⎥⎦
5! ⎝ 6! ⎠ ⎥⎦ ⎣
1 1 ⋅ ⎡⎣(e1 n ) n - 1⎤⎦ 1
= =0 = lim = (e – 1) lim 1 n
∞ n→∞ ⎛ e
n→∞ n ( e1 n - 1) - 1⎞
The correct option is (C) ⎜ 1n ⎟
1 1
⎝ ⎠
lim (cos x + sin x -1) = (e – 1) × 1 = (e – 1).
9. lim (cos x + sin x ) x = e x→0
x
x→0 ( - sin x + cos x ) The correct option is (C)
lim
= e x→0
1
(1 + h)sin (1 + h - [1 + h])
(Using L’Hospital’s Rule) 13. RHL = lim
h→ 0 1+ h -1
= e1 = e
(1 + h)sin (1 + h - 1)
The correct option is (A) = lim
h→ 0 h
2 2 - (cos x + sin x )3 ⎛0 ⎞ sin h
10. lim ⎜⎝ form⎟⎠ = lim(1+ h) =1
x→
p 1 - sin 2 x 0 h→ 0 h
4
-3 (cos x + sin x ) 2 ( - sin x + cos x ) (1 - h)sin (1 - h - [1 - h])
= lim LHL = lim
p -2 cos 2 x h→ 0 1- h -1
x→
4
(Using L’Hospital’s Rule) (1 - h)sin (1 - h)
= lim = –∞
h→ 0 -h
-3 (cos x + sin x )(cos 2 x - sin 2 x )
= lim Since LHL ≠ RHL,
x→
p -2 cos 2 x
x sin ( x - [ x ])
4
\ lim does not exist.
-3 (cos x + sin x )cos 2 x x →1 x -1
= lim
x→
p -2 cos 2 x The correct option is (D)
4
2 sin x - sin 2 x
= lim
3 (cos x + sin x ) 3 ⎛ 1
= ⋅⎜ +
1 ⎞
⎟ =
3 14. f (x) = ∫ x3
dx
x→
p 2 2 ⎝ 2 2⎠ 2
4 d 2 sin x - sin 2 x 2 sin x - sin 2 x
dx ∫
The correct option is (A) ⇒ f ′ (x) = dx =
x3 x3
ln(1 + 2h) - 2 ln(1 + h)
11. lim 2 sin x - sin 2 x ⎛0 ⎞
h→ 0 h2 \ lim f ′( x ) = lim 3 ⎜⎝ form⎟⎠
x →0 x→0 x 0
ln(1 + h) 2 - ln(1 + 2h)
= – lim 2 cos x - 2 cos 2 x ⎛0 ⎞
h→ 0 h2 = lim ⎜⎝ form⎟⎠
x→0 3x 2 0
⎛ (1 + h) 2 ⎞
ln ⎜ ⎟ -2 sin x + 4 sin 2 x ⎛0 ⎞
⎝ 1 + 2h ⎠ = lim ⎜⎝ form⎟⎠
= – lim x→0 6x 0
h→ 0 h2
11.28 Chapter 11

-2 cos x + 8 cos 2 x (log x ) 2 ⎛∞ ⎞


= lim 20. lim ⎜⎝ ∞ form⎟⎠
x→0 6 x →∞ n
x
6 1
= =1 2 log x ⋅
6 x = lim 2 log x ⎛∞ ⎞
lim ⎜⎝ ∞ form⎟⎠
The correct option is (D) x →∞ n x n -1 x →∞ n x n

p 2
15. ∵ < 1, = lim 2 n = 0
4 x →∞ n x
⎛p⎞
\ ⎜ ⎟ =0 The correct option is (B)
⎝ 4⎠
r r
⎛ x⎞ 21. tr = =
⎜⎝ ⎟⎠ r4 + r2 + 1 ( r 2 + 1) 2 - r 2
2
\ lim =0
x → p / 2 ln(sin x ) 1⎛ 1 1 ⎞
= ⎜ - ⎟
The correct option is (C) 2 ⎝ r 2 - r + 1 r 2 + r + 1⎠
16. We know that |cosq | ≤ 1 for all q. 1⎡ 1 1 ⎤
= ⎢ - ⎥
So, if |cos n! px| < 1, 2 ⎣ r ( r - 1) + 1 ( r + 1) r + 1⎦
lim lim (1 + cos 2 m n!p x ) = (1 + 0) = 1 n n
1
m →∞ n→∞
\ ∑ tr = ∑ 2 [ f ( r ) - f ( r + 1)] ,
and if |cosn! px| = 1, r =1 r =1

1 1
lim lim (1 + cos n!p x ) = lim lim (1 + 1 ) = [ f (1) - f ( n + 1)]
2m 2m
where f (r) =
m →∞ n→∞ m →∞ n→∞ r ( r - 1) + 1 2
= lim lim (1 + 1) = 2 1⎡ 1 ⎤ 1
m →∞ n→∞ = ⎢1 - ⎥ → as n → ∞
The correct option is (A) and (B) 2 ⎣ ( n + 1)n + 1⎦ 2
⎡ sin([ x - 3]) ⎤ ⎡ sin( - 4) ⎤ The correct option is (B)
17. LHL = lim ⎢ ⎥ = ⎢ ⎥
x → 0 ⎣ [ x - 3] ⎦ ⎣ -4 ⎦ 22. LHL = lim( -1)[ n - h] = lim( -1) n -1 = (–1)n – 1
-

h→ 0 h→ 0
⎡ sin 4 ⎤ 3p [ n + h]
= ⎢ ⎥ = –1 ∵ p <4< RHL = lim( -1) = lim( -1) n = (– 1)n
⎣ 4 ⎦ 2 h→ 0 h→ 0

⎡ sin[ x - 3] ⎤ ⎡ sin( - 3) ⎤ Since LHL ≠ RHL


RHL = lim ⎢ ⎥ = ⎢ ⎥
x → 0 ⎣ [ x - 3] ⎦
+
⎣ -3 ⎦ \ lim( -1)[ x ] does not exist.
x→n
⎡ sin 3 ⎤ p The correct option is (D)
= ⎢ ⎥ =0 ∵ < 3 < p.
⎣ 3 ⎦ 2 23. Since y = x – 1,
The correct option is (C) \ x = y + 1.
⎛ x2 + 1 ⎞ As (x, y) → (1, 0) along the line y = x – 1, x = y + 1 holds
18. We have, lim ⎜ - ax - b⎟ = 0
x →∞ ⎝ x + 1 ⎠ throughout.
y3 y3
( x 2 + 1) - ( ax + b)( x + 1) \ lim 3 = lim
⇒ lim =0 x →1 x - y 2 - 1 y → 0 ( y + 1)3 - y 2 - 1
x →∞ x +1 y→0
y3 y2 0
x (1 - a) - ( a + b) x - b + 1
2 lim = lim = =0
⇒ lim =0 y→0 y + 2 y + 3y
3 2 y→0 y + 2y + 3
2
3
x →∞ x +1
⇒ 1 – a = 0 and a + b = 0 The correct option is (C)
⇒ a = 1 and b = –1. 1 - 2 + 3 - 4 + 5 - 6 + ... - 2n
24. lim
n→∞
The correct option is (A) n2 + 1 + 4 n2 - 1
⎡ 1 1 1 1 ⎤ [1 + 3 + 5 + 7 + ... + ( 2n - 1)] - ( 2 + 4 + 6 + ... + 2n)
19. lim ⎢ + + + ... + ⎥ = lim
n→∞ ⎣1 ⋅ 2 2 ⋅ 3 3⋅ 4 n( n + 1) ⎦ n→∞ 1 1
n 1+ 2
+n 4- 2
⎡⎛ 1 1 ⎞ ⎛ 1 1 ⎞ ⎛ 1 1 ⎞ ⎛1 1 ⎞⎤ n n
= lim ⎢⎜ - ⎟ + ⎜ - ⎟ + ⎜ - ⎟ + ... + ⎜ - ⎟
n→∞ ⎣ ⎝ 1 2 ⎠ ⎝ 2 3 ⎠ ⎝ 3 4 ⎠ ⎝ n n + 1⎠ ⎥⎦ n n
[2 ⋅1 + ( n - 1) ⋅ 2] - [2 ⋅ 2 + ( n - 1) ⋅ 2]
= lim 2 2
⎡ 1 ⎤
= lim ⎢1 - = 1 – 0 = 1. n→∞ ⎛ 1⎞
n + 1⎥⎦
1
n→∞ ⎣ n ⎜ 1+ 2 + 4 - 2 ⎟
⎝ n n ⎠
The correct option is (A)
Limits 11.29

n n x2 x2
⋅ 2n - 2 ( n + 1) = lim
8
⋅ 2 sin 2 ⋅ 2 sin 2
= lim 2 2 x→0 8
x 4 8
n→∞ ⎛ 1 1⎞
n ⎜ 1+ 2 + 4 - 2 ⎟ ⎛
2 2
⎝ n n ⎠ ⎛ x2 ⎞ x2 ⎞
⎜ sin ⎟ ⎛ 2 ⎞ 2 ⎜ sin ⎟ ⎛ x2 ⎞
2
32 x
n2 - n2 - n = lim 8 ⎜ 24 ⎟ ⎜ ⎟ ⎜ 28 ⎟ ⎜ ⎟
= lim x→0 x
⎜ x ⎟ ⎝ 4⎠ ⎜ x ⎟ ⎝ 8⎠
n→∞ ⎛ 1⎞ ⎝ 4 ⎠ ⎜⎝ ⎟
1
n ⎜ 1+ 2 + 4 - 2 ⎟ 8 ⎠
⎝ n n ⎠
=
1
-n 32
= lim
n→∞ ⎛ 1 1⎞ The correct option is (C)
n ⎜ 1+ 2 + 4 - 2 ⎟
⎝ n n ⎠
28. lim ⎡log n -1( n) ⋅ log n ( n + 1) ⋅ log n +1( n + 2)...log n -1 ( n
k
)⎤⎦
n→ ∞ ⎣
k

-1 -1 -1
= lim = = ⎡ log n log( n + 1) log( n + 2) log( nk ) ⎤
n→∞ 1 1 1+ 2 3 = lim ⎢ ⋅ ⋅
1+ 2 + 4 - 2 ... ⎥
n→∞ ⎢ log( n - 1) log n log( n + 1) log( nk - 1) ⎥⎦
n n ⎣
The correct option is (B) ⎛ log m ⎞
⎜⎝ Using log n m = log n ⎟⎠
⎛ x 4 sin (1 / x ) + x 2 ⎞
25. lim ⎜ ⎟ log nk
x→-∞ ⎝ 1 + | x |3 ⎠ log n ⎛∞ ⎞
= lim = k lim ⎜⎝ ∞ form⎟⎠
n→∞ log( n - 1) n→∞ log( n - 1)
1
- y 4 sin + y 2 1/ n
= lim
y = k lim (Using L’Hospital’s Rule)
y→∞ 1+ y 3
n→∞ 1 / n -1
(Putting x = – y; as x→ – ∞, y→ ∞) ⎛ 1⎞
= k lim ⎜1 - ⎟ = k
n→∞ ⎝ n⎠
⎛ 1⎞
sin
⎜ y⎟ 1 The correct option is (C)
-⎜ ⎟+
⎜ 1 ⎟ y ⎡ 1 1 1 1 ⎤
⎝ y ⎠ -1 + 0 29. lim ⎢ + + + ... + ⎥
= lim = = –1 n→∞ ⎣1 ⋅ 3 3⋅5 5⋅ 7 ( 2n + 1)( 2n + 3) ⎦
y→∞
1+ 3
1 1+ 0
y ⎡ 1 ⎛ 1 1⎞ 1 ⎛ 1 1⎞ 1⎛ 1 1 ⎞⎤
= lim ⎢ ⎜ - ⎟ + ⎜ - ⎟ ... + ⎜ - ⎟
The correct option is (B) n→∞ ⎣ 2 ⎝ 1 3 ⎠ 2 ⎝ 3 5⎠ 2 ⎝ 2n + 1 2n + 3 ⎠ ⎥⎦

2 x + 23 - x - 6 1⎛ 1 ⎞ 1 1
26. lim = lim ⎜⎝1 - ⎟⎠ = (1 – 0) =
x→2 2 - x / 2 - 21 - x n→∞ 2 2n + 3 2 2
( 22 x + 23 - 6 ⋅ 2 x ) / 2 x The correct option is (B)
= lim
x→2 1 2
- nx
2x / 2 2x ⎛ 1x 1 1 1 ⎞
1 + 2 x + 3 x + ... + n x
22 x - 6 ⋅ 2 x + 8 ( 2 x - 4 ) ( 2 x - 2) 30. lim ⎜ ⎟
= lim = lim x →∞ ⎜ n ⎟⎠

x→2 2 x/2
-2 x→2 ( 2 x / 2 - 2)
n

( 2 x / 2 + 2) ( 2 x / 2 - 2) ( 2 x - 2) ⎛ 1y + 2 y + 3 y + ... + n y ⎞ y
= lim = lim ⎜ ⎟
y→0 ⎝ n ⎠
x→2 ( 2 x / 2 - 2)
= lim ( 2 x / 2 + 2) ( 2 x - 2) = (2 + 2) ⋅ (4 – 2) = 8 lim
n ⎛ 1y + 2 y + 3 y + ... + n y ⎞
-1⎟
x→2 y→0
y ⎜⎝ n ⎠
= e
The correct option is (A) ⎛ 1y + 2 y + 3 y + ... + n y - n ⎞
lim
y→0 ⎜ ⎟⎠

8 ⎡ x 2
x x x 2 2 2⎤
= e y
27. lim ⎢1 - cos - cos + cos cos ⎥ ⎡ (1y -1) ( 2 y -1) ( 3 y -1) ( n y -1) ⎤
x→0 x 8 ⎢⎣ 2 4 2 4 ⎥⎦ lim
y→0 ⎢
+ + + ...+ ⎥
= e ⎣ y y y y ⎦

8 ⎡⎛ x2 ⎞ x4 ⎛ x2 ⎞ ⎤
= lim 8 ⎢⎜1 - cos ⎟ - cos ⎜1 - cos ⎟ ⎥ = e (log 1 + log 2 + log 3 + … + log n)
x → 0 x ⎢⎝ 2⎠ 4 ⎝ 2 ⎠ ⎥⎦
⎣ = e log (1 ⋅ 2 ⋅ 3 … n) = n!
8 ⎛ x ⎞⎛
2
x ⎞
2
The correct option is (A)
= lim 8 ⎜1 - cos ⎟ ⎜1 - cos ⎟
x→0 x ⎝ 2 ⎠⎝ 4⎠
11.30 Chapter 11

31. lim (1 + x) (1 + x2) (1 + x4) … (1 + x2n) \


2
lim Sn = –
n→∞ n→∞ 3
(1 - x )(1 + x )(1 + x )(1 + x )...(1 + x )
2 4 2n
The correct option is (B)
= lim
n→∞ 1- x
(1 + a3 ) + 8e1 x ⎛∞ ⎞
(1 - x 2 )(1 + x 2 )(1 + x 4 )...(1 + x 2 n ) 35. We have 2 = lim ⎜⎝ ∞ form⎟⎠ (1)
= lim x → 0 1 + (1 - b3 ) e1 x
n→∞ 1- x
. . . 0 + 8e1 x ( -1 x 2 )
. . . ⇒ 2 = lim
x → 0 0 + (1 - b3 ) e1 x ( -1 x2 )
. . .
(Using L’Hospital’s Rule)
1 - x 4n + 2 1
= lim = for |x| < 1 ⇒ 1–b =4 3
n→∞ 1- x 1- x
⇒ b3 = – 3
The correct option is (B)
⇒ b = (–3)1/3
32. Case I: n is a positive integer
(1 + a3 ) + 8e1 x
xn nx n -1 \ From Eq. (1), 2 = lim
lim = lim x→0 1 + 4e1 x
x →∞ ex x →∞ ex
⇒ 1 + a3 = 2 i.e., a = 1
n ( n - 1) x n - 2 n!
= lim = … = lim Hence a = 1 and b = (–3)1/3.
x →∞ ex x →∞ e x
The correct option is (A)
(Using L’Hospital’s Rule repeatedly)
36. x2 + 4x + 5 = (x + 2)2 + 1 ≥ 1. So, a = 1.
=0
1 - cos 2q 2 sin 2 q
Case II: n is a negative integer. Also, b = lim = lim = 2.
n -m q →0 q 2 q →0 q2
x x n
= lim
lim
x →∞ ex ex
x →∞ \ ∑ ar ⋅ bn - r = bn + abn – 1 + a2 bn – 2 + … + an
r=0
(Putting n = – m, where m is a positive integer)
1 1 ⎡ ⎛ a ⎞ n +1 ⎤ ⎡ ⎛ 1 ⎞ n +1 ⎤
= lim m x = = 0. b n ⎢1 - ⎜ ⎟ ⎥ 2n ⎢1 - ⎜ ⎟ ⎥
x →∞ x e ∞ ⎢⎣ ⎝ b ⎠ ⎥⎦ ⎢⎣ ⎝ 2 ⎠ ⎥⎦
Case III: n = 0 = =
a 1
1- 1-
xn 1 1 b 2
lim x = lim x = = 0.
x →∞ e x →∞ e ∞ 2n +1 ( 2n +1 - 1)
xn = = (2n + 1 – 1)
Hence, lim x = 0 for all values of n. 2n +1
x →∞ e
The correct option is (B)
The correct option is (B)
1 ⋅ 2 + 2 ⋅ 3 + 3 ⋅ 4 + ... + n ( n + 1)
x n + x n -1 + x n - 2 + ... + x 2 + x - n 37. lim
⎛0 ⎞ n→∞ n3
33. lim ⎜⎝ form⎟⎠
x →1 x -1 0
S n ( n + 1) S n 2 + Sn
nx n -1 + ( n - 1) x n - 2 + ... + 2 x + 1 = lim 3
= lim
= lim n→∞ n n→∞ n3
x →1 1
(Using L’Hospital’s Rule) 1 ⎡ n ( n + 1)( 2n + 1) n ( n + 1) ⎤
= lim +
n ( n + 1) n→∞ n3 ⎢⎣ 6 2 ⎥⎦
= n + (n – 1) + … + 2 + 1 =
2 ⎡1 ⎛ 1⎞ ⎛ 1⎞ 1 ⎛ 1 1 ⎞ ⎤
The correct option is (A) = lim ⎢ ⎜1 + ⎟ ⎜ 2 + ⎟ + ⋅ ⎜ + 2 ⎟ ⎥
n→∞ ⎣ 6 ⎝ n⎠ ⎝ n⎠ 2 ⎝ n n ⎠ ⎦
12 + 22 + 32 + ... + r 2
34. tr = 1 1
13 + 23 + 33 + ... + r 3 = ×1×2= .
2 6 3
r ( r + 1)( 2r + 1) ⎛ 2 ⎞
= ⋅⎜ The correct option is (C)
6 ⎝ r ( r + 1) ⎟⎠
log(1 + x + x 2 ) + log(1 - x + x 2 )
2⎛1 1 ⎞ 38. lim
= ⎜ + ⎟ x→0 sec x - cos x

3 r r + 1⎠
log ⎡⎣(1 + x 2 ) 2 - x 2 ⎤⎦
2 ⎡ ⎛ 1 ⎞ ⎛ 1 1⎞ ⎛ 1 1 ⎞ ⎛1 1 ⎞⎤ = lim
\ Sn = ⎢ - ⎜ 1 + ⎟ + ⎜ + ⎟ - ⎜ + ⎟ + … ± ⎜ +
3 ⎣ ⎝ 2 ⎠ ⎝ 2 3⎠ ⎝ 3 4 ⎠ ⎝ n n + 1⎟⎠ ⎥⎦ x→0 (1 - cos 2 x ) cos x

2⎛ 1 ⎞ log(1 + x 2 + x 4 ) ⎛0 ⎞
= ⎜ -1 ± ⎟ = lim ⎜⎝ form⎟⎠
3⎝ n + 1⎠ x→0 sin x tan x 0
Limits 11.31

log[1 + x 2 (1 + x 2 )] 1 3x +1 - 5 x +1 3 ⋅ 3x - 5 ⋅ 5 x
= lim ⋅ x 2 (1 + x 2 ) ⋅ 41. lim = lim
x→0 x 2 (1 + x 2 ) sin x tan x 2
⋅ ⋅x
x →∞ 3x - 5 x x →∞ 3x - 5 x
x x x
⎡ log(1 + x ) ⎤ ⎛ 3⎞
= 1. ⎢as lim = 1⎥ 3⋅ ⎜ ⎟ - 5
⎝ 5⎠ -5
⎣ x → 0 x ⎦ = lim =
x →∞ ⎛ 3⎞
x
-1
The correct option is (A) ⎜⎝ ⎟⎠ - 1
⎛ h⎞ 5
loge e ⎜1 - ⎟ - 1
ln(e - h) - 1 ⎝ e⎠ ⎛ ⎞
39. LHL = lim = lim = 5. ⎜⎝∵ nlim a n = 0, if - 1 < a < 1⎟
h→ 0 | e - h - e | h→ 0 | -h | →∞ ⎠
The correct option is (A)
⎛ h⎞
log e + log ⎜1 - ⎟ - 1 n -1 ⎞
⎝ e⎠ 1⎛
⎜1 + e + e + ... + e
1/ n 2/ n
= lim 42. lim n

h→ 0 h n→∞ n
⎝ ⎠
h h2
( )
n
1 1- e
1/ n
- - 2 - ... 1- e
1 = lim ⋅ = lim
= lim e 2e =– n→∞ ⎛ 1 1 1 ⎞
h→ 0 | e + h - e | e n→∞ n 1 - e1/ n
n ⎜1 - 1 - - ⋅ 2 ...⎟
⎝ n 2! n ⎠
ln (e + h) - 1
RHL = lim 1- e 1- e
h→ 0 |e+h-e| = lim = =e–1
n→∞ 1 1
-1 - ⋅ ... -1
⎛ h⎞ 2! n
log e ⎜1 + ⎟ - 1
⎝ e⎠ The correct option is (C)
= lim
h→ 0 |h|
sin x
⎛ h⎞ 1+
x + sin x x 1+ 0
log e + log ⎜1 + ⎟ - 1 43. lim = lim = = 1.
⎝ e⎠ x →∞ x - cos x x →∞ cos x 1- 0
= lim 1-
h→ 0 h x
h h2 ⎡ sin x ⎛1⎞ ⎤
- 2 + ... ⎢∵ lim = lim y sin ⎜ ⎟ = O × (a finite quantity) ⎥
1
= lim e 2e = ⎢ x →∞ x y →0 ⎝ ⎠
y ⎥
h→ 0 h e ⎢ cos x ⎥
Since LHL ≠ RHL ⎢ = 0. Similarly lim =0 ⎥
⎣ x →∞ x ⎦
ln x - 1
\ lim does not exist. The correct option is (B)
x→e | x - e | n

The correct option is (D) 44. Sn= ∑ ai , lim an = a


n→∞
i =1
40. We have Sn + 1 – Sn = an + 1
x1 = 3, xn + 1 = 2 + xn an + 1 2a
So, lim = lim =0
x2 = 2 + x1 = 2+3 = 5 n → ∞ n( n + 1) n → ∞ n( n + 1)

2
x3 = 2 + x 2 = 2 + 5
The correct option is (A)
\ x1 > x2 > x3
⎡⎛ 1⎞
13 ⎤
45. lim ⎡ n2 - n3 + n⎤ = lim n ⎢⎜ -1 + ⎟ + 11 3 ⎥
It can be easily shown by mathematical induction that the 3

sequence x1, x2, …xn, … is a monotonically decreasing ⎢


n→∞ ⎣ ⎥
⎦ n→∞ ⎢ ⎝ n⎠ ⎥⎦

sequence bounded below by 2. So it is convergent.
⎛1 ⎞
Let lim xn = x. Then ⎜⎝ - 1⎟⎠ + 1
n→∞ n
= lim n ⋅ 23 13
n→∞ ⎛1 ⎞ ⎛1 ⎞
xn + 1 = 2 + xn ⎜⎝ - 1⎟ + 1 - ⎜⎝ - 1⎟
n ⎠ n ⎠
⇒ lim xn + 1 = 2 + lim xn
n→∞ n→∞
⎛ a3 + b3 ⎞
⇒ x= 2+ x ⎜ Using a + b = 2 ⎟
⎝ a - ab + b 2 ⎠
⇒ x2 – x – 2 = 0 1 1 1
= lim = =
⇒ (x – 2)(x + 1) = 0 n→∞ ⎛ 1 23 13
1 + 1 + 1 3
⎞ ⎛ 1 ⎞
⇒ x=2 (∵ xn > 0 ∀n, \ x > 0) ⎜⎝ - 1⎟⎠ + 1 - ⎜⎝ - 1⎟⎠
n n
The correct option is (B)
The correct option is (A)
11.32 Chapter 11

4 3
n5 + 2 - n2 + 1 A h 2 hr - h2
46. lim 50. lim = lim
( 2 hr - h )
n→∞ 5 2 h→0 + 3 h→0 + 3
n4 + 2 - n3 + 1 P
8 2
+ 2 hr
2 1
n5 4 4 1 +
5
- n2 3 3 1 + 2 h ⋅ h 2r - h
= lim n n = lim
( )
h→0 3
n→∞ 4 5 2 1 8⋅ h ⋅ h 2r - h + 2r
n 5 1 + 4 - n3 2 2 1 + 3
n n
2r - h
n5 4 4 2 n2 3 = lim
( )
1 3
32
1+ 5 - 3 2 3 1+ 2 h→0
8 2r - h + 2r
n
= lim 4 5 n n n
n→∞ n
5 1+
2 n3 2 2 1 2r 1
32
- 1+ 3 = =
n 4 n3 2
n n
( )
3
8 2 2r 128r
(Dividing the numerator and denominator by the highest
The correct option is (B)
power n3/2)
1 2 1 3 1 ⎛ 1 - cos 2( x - 2) ⎞
4 1+ - 1+ 2 51. lim ⎜ ⎟
n 14
n5 n5 6 n = 0 - 0 = 0. x → 2⎝ x-2 ⎠
= lim
n→∞ 1 5 2 1 0 -1 2 | sin( x - 2) |
7 10
1+ 4 - 2 1+ 3 = lim
n n n x→2 x-2
The correct option is (B) which doesn’t exist as LHL = - 2 whereas
47. Minimum power in numerator on x is 3. So n = 3. RHL = 2
The correct option is (C) The correct option is (B)
2 x +33 x +55 x x x x x
48. lim 52. Required limit= lim cos cos 2 cos 3 ... cos n
2 n→∞2 2 2
x→∞ 3x - 2 + 2 x - 3 3

1 ⎧ x x ⎛ x x ⎞⎫
= lim
x ⎨⎩
2 x +33 x +55 x cos ... cos n - 1 ⎜ 2 sin n cos n ⎟ ⎬
= lim n→∞ 2 2 ⎝ 2 2 ⎠⎭
x→∞
2 sin n
2 3 3 2
x 3- + x 3 2-
x x 1 ⎧ x ⎛ x x ⎞⎫
= lim
x ⎨⎩
cos ... ⎜ 2 cos n - 1 sin n - 1 ⎟ ⎬
3 5 n→∞ 2 2 ⎝ 2 2 ⎠⎭
1/ 6
+ 3/10 2+ 2 sin n
= lim x x 2
x→∞ 2 1 3 …………………………………………………………..
3 - + 1/ 6 3 2 -
x x x …………………………………………………………..
(Dividing the numerator and denominator by the highest ⎛ 1x x⎞
power x1/2) = lim 2 cos sin ⎟
n→∞ x ⎜⎝ n 2 2 ⎠
2 2 sin n
= 2
3 sin x
The correct option is (A) = lim
n→∞ n ⎛ x⎞
2 sin ⎜ n ⎟
⎝2 ⎠
x 3
z 2 - ( z - x )2
49. lim ⎧ ⎛ x⎞ ⎫
x→0 3
( 8 xz - 4 x 2 + 3 8 xz ) 4 ⎪⎪ ⎜⎝ n ⎟⎠ ⎪⎪
sin x 2 sin x
3 = lim ⎨ ⎬ =
= lim
x 2 xz - x 2 x n → ∞
⎪ sin ⎛⎜ ⎞⎟ ⎪
x x
x→0 ( 3 x 3
8z - 4 x + 3 8z 3
x )4 ⎪⎩ ⎝ 2n ⎠ ⎪⎭

x4 3 3
2z - x The correct option is (B)
= lim
( )
x→0 4 53. The (r + 1)th term of the series is
x4 3 3
8z - 4 x + 3 8z n-r
3 tr + 1 = ( r + 1) ∑ k
2z
= k =1

( )
4
3
2 8z ⇒ tr + 1 = (r + 1){1 + 2 + 3 + … (n – r) terms}
1 1
= ⇒ tr + 1 = ( r + 1) ( n - r )( n - r + 1)
223 3 ⋅ z 2
1
The correct option is (B) ⇒ tr + 1 = ( r + 1)( n2 - rn + n - rn + r 2 - r )
2
Limits 11.33

1 According to Squeeze Principle or Sandwich Theorem, we


⇒ tr + 1 = ( r + 1)( r 2 - (1 + 2n)r + n2 )
2 have
x
1 L= .
⇒ tr + 1 = ( r 3 - 2nr 2 + ( n2 - 2n - 1)r + n2 ) 3
2
n -1 The correct option is (C)
Now, S= ∑ tr + 1 55. tan 2q =
sin 2q
r=0
cos 2q
1 n 3
\ S= ∑ {r - 2nr 2 + ( n2 - 2n - 1)r + n2}
2 r =1 ⇒
1
=
2 sin q cos q
cot 2q cos 2 q - sin 2 q
2
1 ⎡ ⎧ n( n + 1) ⎫ ⎧1 ⎫
⇒ S= ⎨ ⎬ - 2n ⎨ n ( n + 1)( 2n + 1)⎬ cos 2 q - sin 2 q
2 ⎢⎣ ⎩ 2 ⎭ ⎩ 6 ⎭ ⇒ cot 2q =
2 sin q cos q
⎧1 ⎫ ⎤ cos 2 q sin 2 q
+ ( n2 - 2n - 1) ⎨ n( n + 1)⎬ + n2 ( n)⎥ ⇒ 2cot 2q = -
⎩2 ⎭ ⎦ sin q cos q sin q cos q
Solving and rearranging, we have, ⇒ 2cot 2q = cot q – tan q
1 4 ⇒ tan q = cot q – 2 cot 2q
S= {n - 11n3 - 19n2 + 6 n} (1)
24 Now, tan q = cot q – 2 cot 2q
S 1 ⎛ n4 - 11n3 - 19n2 + 6 n ⎞
\ lim 4 = lim ⎜ ⎟ 1 q 1 q
n→∞ n n → ∞ 24 ⎝ n4 ⎠ ⇒ tan = cot - cot q
2 2 2 2
1 ⎛ 11 19 6 ⎞ 1 q 1 q 1
= lim ⎜1 - - 2 + 3 ⎟ ⇒ tan 2 = 2 cot - cot q
24 n → ∞ ⎝ n n n ⎠ 2 2
2 2 2 2
n -1⎧ n-r ⎫⎪ ………
1 ⎪ 1
\ lim
n→∞ 4 ∑ ⎨( r + 1) ∑ k ⎬ = 24
………
r = 0⎪
n
⎩ ⎪
k =1 ⎭
1 q 1 q 1 q
The correct option is (A) ⇒ n n
= n cot n - n - 1 cot n - 1
tan
2 2 2 2 2 2
1 ⎛ n 2 ⎞ 1 q
54. Let L = lim ⎜ ∑ [k x ]⎟ ⇒ S = - 2 cot 2q + n cot n
n → ∞ n3 ⎝ ⎠ 2 2
k =1
⎛ 1 q 1 q 1 q⎞
= lim ⎜ tan q + tan + 2 tan 2 +… + n tan n ⎟
Since k2x – 1 ≤ [k2x] < k2x n→∞ ⎝ 2 2 2 2 2 2 ⎠
n n n
⎛ 1 q⎞
⇒ ∑ ( k 2 x - 1) ≤ ∑ [k 2 x] < ∑ k 2 xa \ lim S = lim ⎜ - 2 cot 2q + n cot n ⎟
n→∞ n→ ∞⎝ 2 2 ⎠
k =1 k =1 k =1

⎛ n ⎞ n n ⎛ n ⎞ ⎡ q ⎤
⇒ x ⎜ ∑ k 2 ⎟ - ∑ (1) ≤ ∑ [k 2 x ] < x ⎜ ∑ k 2 ⎟ 1 ⎢ 2n ⎥
⎝k =1 ⎠ k =1 ⎝k =1 ⎠ = - 2 cot 2q + lim ⎢ ⎥
k =1 n→∞q ⎢ q
tan n ⎥
⎢⎣ 2 ⎥⎦
xn( n + 1)( 2n + 1) n
xn ( n + 1)( 2n + 1)
⇒ - n ≤ ∑ [k 2 x ] < = - 2 cot 2q +
1
6 k =1 6 q
3
Dividing throughout by n , we have The correct option is (B)
xn ( n + 1)( 2n + 1) [k 2 x ] xn ( n + 1)( 2n + 1) 4 3n - 2 - 9 n + 1
n
1
6n 3
-
n 2
≤ ∑ n 3
<
6n3 56. lim
n→∞ 82 n - 1 - 9 n - 1
k =1

x⎛ 1⎞ ⎛ 1⎞ 1 n
[k 2 x ] x ⎛ 1⎞ ⎛ 1⎞ 4 - 2 ⋅ 64 n - 9 ⋅ 9n
⇒ ⎜1 + ⎟ ⎜2 + ⎟⎠ - 2 ≤ ∑ 3 < ⎜⎝1 + ⎟ ⎜2 + ⎟ = lim
8-1 ⋅ 64 n - 9-1 ⋅ 9n
6⎝ n⎠ ⎝ n n k =1 n 6 n⎠ ⎝ n⎠ n→ + ∞
n
Taking limits as n→ ∞, we get ⎛ 9⎞
4- 2 - 9 ⎜ ⎟
⎝ 64 ⎠
⎧x ⎛ 1⎞ ⎛ 1⎞ 1 ⎫ x⎛ 1⎞ ⎛ 1⎞ = lim
lim ⎨ ⎜1 + ⎟ ⎜ 2 + ⎟ - 2 ⎬ ≤ L < lim ⎜1 + ⎟ ⎜ 2 + ⎟ n→ + ∞ ⎛ 9⎞
n
n → ∞ ⎩6 ⎝ n⎠ ⎝ n⎠ n ⎭ n→∞ 6 ⎝ n⎠ ⎝ n⎠ 8-1 - 9-1 ⎜ ⎟
⎝ 64 ⎠
1
Since, as n→ ∞, we have → 0 4- 2 - 0 1
n = -1 =

x
≤L<
x 8 -0 2
3 3
11.34 Chapter 11

n
⎛ 64 ⎞ 1
4 -2 ⎜ ⎟ - 9 = lim
⎝ 9⎠ x →∞ ⎡⎛ 1 1 k2 1 k3 1 k4 1⎞
= lim ⎢⎜ 5 + k ⋅ 4 + ⋅ + ⋅ + ⋅
n→- ∞ ⎛ 64 ⎞
n
⎢⎣⎝ x x 2! x 3 3! x 2 4! x ⎟⎠
8-1 ⎜ ⎟ - 9 -1
⎝ 9⎠
k5 ⎛ k6 ⎞⎤
0-9 + + ⎜ x + ...⎟ ⎥
= = 81 5! ⎝ 6! ⎠ ⎥⎦
0 - 9 -1
1
Hence, limit does not exist. = =0

The correct option is (C)
The correct option is (C)
x - ai
57. We have, Ai = , i = 1, 2, …, n r r
| x - ai | 60. tr = =
and, a1 < a2 < … an – 1 < an r + r +1
4 2
( r + 1) 2 - r 2
2

Let x be in the left neighbourhood of am 1⎡ 1 1 ⎤


= -
Then, x – ai < 0 for i = m, m + 1, … n 2 ⎢⎣ r 2 - r + 1 r 2 + r + 1⎥⎦
and x – ai > 0 for i = 1, 2, …, m – 1 1⎡ 1 1 ⎤
x - ai = ⎢ - ⎥
\ Ai = = –1, for i = m, m + 1, …, n 2 ⎣ r ( r - 1) + 1 ( r + 1) r + 1⎦
- ( x - ai )
x - ai n n
1
and, Ai =
x - ai
= 1, for i = 1, 2, …, m – 1 \ ∑ tr = ∑ 2 [ f ( r ) - f ( r + 1)] ,
r =1 r =1
Similarly, if x is in the right neighbourhood of ai 1
where, f (r) =
Then, x – ai< 0 for i = m + 1, …, n r ( r - 1) + 1
and x – ai> 0 for i = 1, 2, …, m 1
= [ f (1) - f ( n + 1)]
x - ai 2
\ Ai = = –1 for i = m + 1, …n
- ( x - ai ) 1⎡ 1 ⎤ 1
x - ai = ⎢1 - ⎥ → as n→ ∞
and, Ai = = 1 for i = 1, 2, …, m 2 ⎣ ( n + 1)n + 1⎦ 2
x - ai
The correct option is (B)
Now, lim ( A1 A2 ... An ) = (–1)n – m + 1 nx
x → am-
⎡11/x + 21/x + 31/x + ... + n1/x ⎤
and, lim ( A1 A2 ... An ) = (–1) n–m 61. lim ⎢ ⎥
x →∞ ⎢ n ⎥⎦
x → am+ ⎣
n
Hence, lim ( A1 A2 ... An ) does not exist.
x → am ⎡1y + 2 y + 3 y + ... + n y ⎤ y
= lim ⎢ ⎥
The correct option is (D) y→0 ⎢ n ⎥⎦

sin x n ⎡1y + 2 y + 3 y + ... + n y ⎤
58. We know that lim → 1– lim ⎢ -1⎥
x→0 x = e y⎣
y→0
n ⎦
x
lim → 1+ lim
⎡1y + 2 y + 3 y + ... + n y - n ⎤
x → 0 sin x y→0 ⎢ ⎥
= e ⎣ y ⎦
⎡ x ⎤ ⎡ sin x ⎤ ⎡ (1y -1) ( 2 y -1) ( 3 y -1) ( n y -1) ⎤
\ lim ⎢100 ⎥ + lim 99 = 100 + 98 = 198. lim
y→0 ⎢
+ + + ...+ ⎥
x→0⎣ sin x ⎦ x → 0 ⎢⎣ x ⎥⎦ = e ⎣ y y y y ⎦

The correct option is (B) = e (log 1 + log 2 + log 3 + … + log n)


x5 = e log (1 ⋅ 2 ⋅ 3 …. n) = n!
59. lim The correct option is (A)
x →∞ 5 x
62. lim (1 + x) (1 + x2) (1 + x4) … (1 + x2n)
x5 n→∞
= lim
x →∞ e x log 5 (1 - x )(1 + x )(1 + x 2 )(1 + x 4 )...(1 + x 2 n )
= lim
n→∞ 1- x
x5
= lim , where k = log 5 (1 - x 2 )(1 + x 2 )(1 + x 4 )...(1 + x 2 n )
x →∞ e kx
= lim
x5
n→∞ 1- x
= lim . . .
x →∞ ⎛ k 2 x 2 k 3 x 3 k 4 x 4 x 5k 5 k 6 x 6 ⎞
⎜1 + kx + 2! + 3! + 4! + 5! + 6! + ...⎟ . . .
⎝ ⎠ . . .
Limits 11.35

1 - x 4n + 2 1 4
n5 + 2 - n 2 + 1
3
= lim = for |x| < 1 66. lim
n→∞ 1- x 1- x n→∞ 5 2
n4 + 2 - n3 + 1
The correct option is (B)
2 1
63. Case I: n is a positive integer n5 4 4 1 +
5
- n2 3 3 1 + 2
n n
xn nx n -1 = lim
lim = lim n→∞ 4 5 2 1
x →∞ e x x →∞ ex n 5 1+ 4 - n 2 1+ 3
3 2
n n
n ( n - 1) x n - 2 n!
= lim = … = lim n5 4 4 2 n2 3 1
x →∞ ex x →∞ e x
1+ 5 - 3 2 3 1+ 2
32
[Using L’Hospital’s rule repeatedly] n
= lim 4 5 n n n
=0
n→∞ n
5 1+
2 n3 2 2 1
32
- 1+ 3
Case II: n is a negative integer. n n 4 n3 2 n
[Dividing the numerator and denominator by the highest
xn x -m
lim x
= lim power n3/2]
x →∞ e ex
x →∞

[Putting n = – m, where m is a positive integer] 1 4 2 1 1


1+ 5 - 5 6 3 1+ 2
n 14
n n n 0-0
= lim m x =
1 1
=0 = lim = = 0.
x →∞ x e ∞
n→∞ 1 5 2 2 1 0 -1
1 + - 1 +
Case III: n = 0 n7 10 n4 n3
xn 1 1 The correct option is (B)
lim x = lim x = =0
x →∞ e x →∞ e ∞
x 3
z 2 - ( z - x )2
xn 67. lim
Hence, lim x = 0 for all values of n. x→0 3
( 8 xz - 4 x 2 + 3 8 xz ) 4
x →∞ e
The correct option is (B) 3
x 2 xz - x 2
64. x2 + 4x + 5 = (x + 2)2 + 1 ≥ 1. = lim
x→0 ( 3 x 3
8z - 4 x + 3 8z 3
x )4
So, a = 1
1 - cos 2q 2 sin 2 q x4 3 3
2z - x 3
2z
Also, b = lim = lim =2 = lim 4
= 4
q →0 q2 q →0 q 2 x→0 4 3 ⎡3 ⎤ ⎡ ⎤
⎣ 8z - 4 x + 8z ⎦
3 3
n
x ⎣2 8z ⎦
\ ∑ ar ⋅ bn - r = bn + abn – 1 + a2 bn – 2 + … + an =
1
r=0
n +1 ⎤ 223 3 ⋅ z
⎡ ⎛ a⎞ ⎡ ⎛ 1 ⎞ n +1 ⎤
b n ⎢1 - ⎜ ⎟ ⎥ 2n ⎢1 - ⎜ ⎟ ⎥ The correct option is (B)
⎢⎣ ⎝ b ⎠ ⎥⎦ ⎣⎢
⎝ 2⎠ ⎥

= =
a 1 h 2 hr - h2
1- 1- A
b 2 68. lim 3
= lim 3
h→0 +
h→0 +

8 ⎡ 2hr - h2 + 2hr ⎤
P
n +1 n +1
2 (2 - 1) ⎣⎢ ⎦⎥
= = (2n + 1 – 1)
2n +1
h ⋅ h 2r - h
The correct option is (B) = lim 3
h→0
8⋅ h ⋅ h ⎡⎣ 2r - h + 2r ⎤⎦
log(1 + x + x ) + log(1 - x + x )
2 2
65. lim
x→0 sec x - cos x 2r - h
= lim
log ⎡⎣(1 + x 2 ) 2 - x 2 ⎤⎦ h→0 3
8 ⎣⎡ 2r - h + 2r ⎤⎦
= lim
x→0 (1 - cos 2 x ) cos x
2r 1
log(1 + x 2 + x 4 ) ⎛0 ⎞ = =
( )
3
= lim form⎟ 128r
x→0 sin x tan x ⎝⎜ 0 ⎠ 8 2 2r
The correct option is (B)
log(1 + x 2 (1 + x 2 )) 1
= lim ⋅ x 2 (1 + x 2 ) ⋅
x→0 x (1 + x )
2 2 sin x tan x 2
⋅ ⋅x 69. lim
(
cos x + p
6 = lim )
cos p + z
2 ( )
x x x →p /3 (1 - 2 cos x ) 2/3
( )
z→ 0 ⎡ 2/3
⎛ log(1 + x ) ⎞ 1 - 2 cos p +z⎤
= 1. ⎜ as lim = 1⎟ ⎣ 3 ⎦
⎝ x→0 x ⎠
The correct option is (A) [putting x –p/3 = z]
11.36 Chapter 11

- sin z 1 -1 1
= lim =- ⋅ =
(1 - cos z + )
z→ 0 2/3
3 sin z 2 2 2

RHL = lim
1 sin 2
x

1 ( )
⎛ z⎞ ⎛ z⎞ x→ 0 x
-2 sin ⎜ ⎟ cos ⎜ ⎟
+
2 1 + cos x
⎝ 2⎠ ⎝ 2⎠ 2
= lim 2/3 2/3 1 1 1
z→ 0 ⎡ ⎛ z⎞⎤ ⎡ ⎛ z⎞ ⎛ z⎞⎤ = ⋅ =
⎢ 2 sin ⎜⎝ 2 ⎟⎠ ⎥ ⎢sin ⎝⎜ 2 ⎟⎠ + 3 cos ⎜⎝ 2 ⎟⎠ ⎥ 2 2 2
⎣ ⎦ ⎣ ⎦
Hence, limit does not exist.
1/3
⎡ ⎛ z⎞⎤ ⎛ z⎞ The correct option is (C)
-21/3 ⎢sin ⎜ ⎟ ⎥ cos ⎜ ⎟
⎣ ⎝ 2 ⎠ ⎦ ⎝ 2⎠ -21/3 ⋅ 0 ⋅1 x + 7 - 3 2x - 3
= lim = =0
( )
z→ 0 ⎡ 2/3 2/3 73. lim
⎛ z⎞ ⎛ z⎞⎤ 3 x→ 2 3 x + 6 - 2 3 3x - 5
⎢sin ⎜⎝ 2 ⎟⎠ + 3 cos ⎜⎝ 2 ⎟⎠ ⎥
⎣ ⎦ ⎡
( x + 7) - 9( 2 x - 3) a2 - b2 ⎤
The correct option is (C) = lim × ⎢ using a - b = ⎥
x→ 2 x + 7 + 3 2x - 3 ⎢⎣ a + b ⎥⎦
ln ( 2 - cos 2 x ) ln {1 + (1 - cos 2 x )}
70. lim = lim ( x + 6) 2/3 + 2( x + 6)1/3 (3 x - 5)1/3 + 4(3 x - 5) 2/3
x→ 0 ln2 (sin 3 x + 1) x→ 0 ln2 (1 + sin 3 x )
( x + 6) - 8 (3 x - 5)
1 - cos 2 x 2x2 2
= lim = lim = ⎡
x→ 0 (sin 3 x ) 2 x→ 0 (3 x ) 2 9 a3 - b3 ⎤
⎢ using a - b = 2 ⎥
The correct option is (A) ⎢⎣ a + ab + b 2 ⎥⎦
1 - cos (cx 2 + bx + a) -17 ( x - 2)
71. lim = lim
x → 1/a (1 - xa ) 2 x→ z x + 7 + 3 2x - 3
1 - cos (cx 2 + bx + a) (cx 2 + bx + a) 2 ( x + 6) 2/3 + 2( x + 6)1/3 (3 x - 5)1/3 + 4(3 x - 5) 2/3
= lim ⋅ ×
x → 1/a (cx 2 + bx + a) 2 (1 - xa ) 2 -23 ( x - 2)
1 - cos z ( ay 2 + by + c) 2 -17 82/3 + 2.81/3 + 4 17 12 34
= lim ⋅ lim = ⋅ = ⋅ =
z→ 0 z 2 y →a y ( y - a)
2 2
9+3 1 -23 6 23 23
[putting cx2 + bx+ a = z and x = 1/y] The correct option is (B)
1 a2 ( y - a )2 ( y - b )2 ( 2m + x )1/m - ( 2n + x )1/n
= lim 74. lim
2 y→ a y 2 ( y - a )2 x→ 0 x
[If a, b are roots of ax + bx+ c = 0 then ax + bx+ c = a
2 2
( 2m + x )1/m - 2 ( 2n + x )1/n - 2
= lim - lim
(x – a) (x – b)] x→ 0 x x→ 0 x
a 2 (a - b ) 2 a2 ⎡ a-2 b-2
= = (a + b ) 2 - 4ab ⎤⎦ = lim m - lim n
2a 2
2a 2 ⎣ a → 2 a - 2m b → 2 b - 2n

⎛ b 2 4c ⎞ [Putting 2m+ x = am and 2n+ x = bn]


a 2
b 2 - 4 ac
= ⎜ - ⎟ = 1 1
2a 2 ⎝ a 2 a⎠ 2a 2 = -
The correct option is (A) m 2m - 1 n 2n - 1
The correct option is (C)
1 - cos x (cos q ) x - (sin q ) x - cos 2q
72. lim 75. lim
x→ 0 x x→ 4 x-4
1 - cos x 1 (cos q ) y + 4 - (sin q ) y + 4 - (cos 4 q - sin 4 q )
= lim ⋅ = lim
x→ 0 x y→ 0 y
1 + cos x
[Putting x – 4 = y and cos 2q = cos4 q – sin4 q]

= lim
2 sin x ( 2) ⋅ 1 ⎡ (cos q ) y - 1⎤ ⎡ (sin q ) y - 1⎤
2(x )
x→ 0 = lim cos 4 q ⎢ ⎥ - sin q
4
⎢ ⎥
2 1 + cos x y→ 0 ⎢⎣ y ⎥⎦ ⎢⎣ y ⎥⎦
Now, we have, = cos4 q lncos q – sin4 q lnsin q

LHL = lim
-1 sin 2

x

1 ( ) The correct option is (A)
x→ 0 x 2 -
2 1 + cos x
Limits 11.37

1/ x 1 ⎛ x - 1 + cos x ⎞ [ x ] + [2 x ] + [3 x ] + ... + [nx ]


⎛ x - 1 + cos x ⎞ ln ⎜
⎝ ⎟⎠ 79. Let f (x) =
76. lim ⎜ ⎟⎠ = lim e x x
1 + 2 + 3 + ... + n
x→ 0 ⎝ x x→ 0
Now, we have,
1 ⎛ 1 - cos x ⎞ x + 2 x + 3 x + ... + nx
Now, lim ln ⎜1 - ⎟⎠ f (x) ≤ =x
x→ 0 x ⎝ x 1 + 2 + 3 + ... + n
⎛ cos x - 1⎞ ( x - 1) + ( 2 x - 1) + (3 x - 1) + ... + ( nx - 1)
ln ⎜1 + ⎟⎠ and, f (x) >
cos x - 1 ⎝ x 1 + 2 + 3 + ... + n
= lim ⋅
x→ 0 x 2 cos x - 1 x Sn - n 2
= =x-
x Sn n +1
-2 sin 2 ( x/2) ln (1 + z ) -1 [∵ x - 1 ≤ [ x ] < x ∀x ∈ R]
= lim 2
⋅ lim =
x→ 0 4( x/2) z→ 0 z 2 Thus, we have,
⎡ cos x - 1 2
< f (x) ≤ x
⎢ Putting z = , we can see that x–
⎣ x n +1
Now, we have,
cos x - 1 ⎛ cos x - 1⎞ -1 ⎤
lim = lim x ⎜ ⎟⎠ = 0 ⋅ = 0 ⎥ 2
x→ 0 x n → 0 ⎝ x 2
2 ⎦ lim x - = x and lim x = x
n→∞ n +1 n→∞

Hence, the required limit is e–1/2. Hence, by Sandwich Theorem, we have


The correct option is (B) lim f ( x ) = x
x 1/ y n→∞
⎡ e ⎤ ⎡ e ⎤ The correct option is (A)
77. lim ⎢ ⎥ = lim ⎢ ⎥
⎣( ) ⎦ y → 0 ⎢⎣ (1 + y ) ⎥⎦
( )
x
x →∞ ⎢ 1 + 1/ x ⎥ 1/ y
1 1 ⎛ 1 1 ⎞
-
80. lim n2 x1/n - x n + 1 = lim n2 ⋅ x n + 1 ⎜⎝ x n n + 1 - 1⎟⎠
n→∞ n→∞
1 ⎡ ⎤

( )
e
ln ⎢ ⎥
y ⎢⎣ (1 + y )1/ y ⎥⎦ 1 1
= lim e
y→ 0 = lim x n + 1 x n ( n + 1) - 1 n2
n→∞
Now, we have, 1
1 1
ln e - ln (1 + y ) n +1 x n ( n + 1) - 1 n2
1 ⎡ e ⎤ y = lim x ⋅ ⋅
lim ln ⎢ ⎥ = lim n→∞ 1
n ( n + 1)
y→ 0 y ⎢⎣ (1 + y ) ⎥⎦
1/ y y→ 0 y n ( n + 1)

= 1 · lnx · 1 = lnx
y - ln (1 + y )
= lim 2 The correct option is (C)
y→ 0 y
81. We have,
⎛ y 2 y3 y 4 ⎞ 1/ x
y-⎜y- + - + ...⎟ ⎡ f ( x) ⎤
⎝ ⎠ lim 1 + x + = e3
x→ 0 ⎢ x ⎥⎦
2 3 4
= lim ⎣
y→ 0 y2 ln [1 + x + g ( x )]
⎡ f ( x) ⎤
1 y y 1 2 ⇒ lim e x = e3 ⎢ Putting x = g ( x )⎥
= lim - + - ... = x→ 0 ⎣ ⎦
y→ 0 2 3 4 2 ln [1 + x + g ( x )]
⇒ lim =3
Hence, the required limit is e1/2. x→ 0 x
The correct option is (C) Since, the denominator approaches zero, the numerator
should also approach zero for a finite limit to exist.
⎡ a sin x ⎤ ⎡ b tan x ⎤
78. lim ⎢ +
x ⎥⎦ ⎢⎣ x ⎥⎦
Thus, we have,
x→ 0 ⎣
lim g ( x ) = 0
x→ 0
⎡ sin x sin x
⎢as x → 0, x → 1 but x < 1 Now, using L’Hospital’s rule, the above equation reduces to

1 + g ′( x )
tan x tan x ⎤ lim =3
while → 1 but > 1⎥ x→ 0 1 + x + g( x)
x x ⎦ i.e., 1 + g′(0) = 3 ⇒ g′(0) = 2
= (a – 1) + b Hence, we have,
=a+b–1 1/ x ln [1 + g ( x )]
⎡ f ( x) ⎤
The correct option is (B) lim 1 + = lim e x
x→ 0 ⎢
⎣ x ⎥⎦ x→ 0
11.38 Chapter 11

g ′( x ) 85. We have,
= lim e1 + g ( x ) = e g ′( 0 )
( )
2 2 2 cos 2 x
x→ 0 lim 11/cos x
+ 21/cos x
+ ... + n1/cos x
x → p /2
= e2
( ) ⎡ 1 ⎤
1/n
The correct option is (B) = lim 1y + 2 y + ... + n y ⎢ Putting cos 2 x = y ⎥
y→∞ ⎣ ⎦
x x
82. Let y = x + =x+ ⎡⎛ 1 ⎞ y ⎛ 2 ⎞ y
1/ y
⎛ n⎞ ⎤
y
x y = lim n ⎢⎜ ⎟ + ⎜ ⎟ + ... + ⎜ ⎟ ⎥
x+ y →∞ ⎢ ⎝ n ⎠ ⎝ n⎠ ⎝ n⎠ ⎥
x ⎣ ⎦
x+
... ∞ = n (0 + 0 + … + 1) = n
i.e., y2 – xy– x =0 The correct option is (A)
n
x ± x2 + 4 x ⎛ 3⎞
i.e., y= 86. lim
n→∞
∑ cot -1 ⎜⎝ r 2 + 4 ⎟⎠
2 r =1
We can see y is a positive quantity for positive x, therefore n ⎡ ( r - 1/2) ( r + 1/2) + 1⎤
x+ x +4 x 2 = lim
n→∞
∑ cot -1 ⎢⎢ ( r + 1/2) - ( r - 1/2) ⎥⎥
y= r =1 ⎣ ⎦
2 n
⎛ 1⎞ ⎛ 1⎞
Hence, the required limit is = lim
n→∞
∑ cot -1 ⎜⎝ r - 2 ⎟⎠ - cot -1 ⎜⎝ r + 2 ⎟⎠
x 2x r =1
= lim = lim
x →∞ y x →∞
x + x2 + 4 x ⎡ -1 ⎛ ab + 1⎞ ⎤
⎢∵cot ⎜ ⎟ = cot -1 a - cot -1 b ⎥
2 2 ⎣ ⎝ b-a⎠ ⎦
= lim = =1
x →∞
1 + 1 + 4x -3/2 1 + 1 ⎡ ⎛ 1⎞ ⎛ 3⎞ ⎛ 3⎞ ⎛ 5⎞
= lim ⎢cot -1 ⎜ ⎟ - cot -1 ⎜ ⎟ + cot -1 ⎜ ⎟ - cot -1 ⎜ ⎟ +
The correct option is (A) n→∞ ⎣ ⎝ 2⎠ ⎝ 2⎠ ⎝ 2⎠ ⎝ 2⎠
83. We have, ⎛ 1⎞ ⎛ 1⎞ ⎤
cos x - (cos x )cos x ... + cot -1 ⎜ n - ⎟ - cot -1 ⎜ n + ⎟ ⎥
lim ⎝ 2⎠ ⎝ 2⎠ ⎦
x → 0 1 - cos x + ln (cos x )
⎡ ⎛ 1⎞ ⎛ 1⎞ ⎤
⎡ 1 - (cos x )cos x -1 ⎤ = lim ⎢cot -1 ⎜ ⎟ - cot -1 ⎜ n + ⎟ ⎥
n→∞ ⎣ ⎝ 2⎠ ⎝ 2⎠ ⎦
= lim cos x ⎢ ⎥
x→ 0 ⎢⎣1 - cos x + ln (cos x ) ⎥⎦ ⎛ 1⎞
1 - (1 + t )t = cot–1 ⎜⎝ 2 ⎟⎠ – 0 = tan–1 2
= lim [Putting cos x – 1 = t]
t → 0 ln (1 + t ) - t
The correct option is (B)
t 3 (t - 1) t 4 (t - 1) (t - 2) ⎛ p⎞
t2 + + + ... 87. Here, sin x + cos x = 2 sin ⎜ x + ⎟
2! 3! ⎝ 4⎠
= lim 2 3 4
=2
t→ 0 t t t
- + - ... 5p ⎛ p⎞
2 3 4 For x → + h, 2 sin ⎜ x + ⎟ → - 2 ,
4 ⎝ 4⎠
The correct option is (C) But greater than – 2
84. We have,
⎡ ⎛ p⎞⎤
(tan x ) tan x - tan x \ ⎢ 2 sin ⎜⎝ x + 4 ⎟⎠ ⎥ = –2 (1)
lim ⎣ ⎦
x → p /4 ln (tan x ) - tan x + 1
5p ⎛ p⎞
tt - t ⎛ 0⎞ Also, for x → - h, 2 sin ⎜ x + ⎟ → - 2 , but greater
= lim [Putting tan x = t] 4 ⎝ 4⎠
⎜ ⎟
t → 1 ln t - t + 1 ⎝ 0 ⎠ than – 2
⎡ ⎛ p⎞⎤
t t (1 + ln t ) - 1 ⎛ 0 ⎞ \ ⎢ 2 sin ⎜⎝ x + 4 ⎟⎠ ⎥ = –2 (2)
= lim ⎜⎝ ⎟⎠ ⎣ ⎦
t→ 1 1 0
-1 From (1) and (2), we get
t
⎛ 1⎞
t t (1 + ln t ) 2 + t t ⎜ ⎟ lim [sin x + cos x ]
⎝t⎠ 1+1 x → 5p /4
= lim = = –2
t→ 1 - 1 -1 = lim - 2 = –2
x→ 5p /4
t2
The correct option is (B)
The correct option is (A)
Limits 11.39

88. lim lim ⎛1 a ⎞


m →∞ n→∞ = lim ⎜ + n ⎟
n→∞ ⎝ n! n! ⎠
⎧⎪1 + n 1n + 2n + n 2n + 3n + ... + n ( m - 1) n + m n ⎫⎪
⎨ ⎬ ⎡1 1 a ⎤
m2 = lim ⎢ + + n -1 ⎥ [using (1)]
⎪⎩ ⎪⎭ n→∞ ⎣ n! ( n - 1)! ( n - 1)! ⎦
= lim
m→∞ ⎡1 1 1 1 1 a⎤
= lim ⎢ + + + ... + + + 1 ⎥
⎧ n n n ⎫
n→∞ ⎣ n! ( n - 1)! ( n - 2)! 2! 1! 1! ⎦
⎪ ⎛ 1⎞ ⎛ 2⎞ ⎛ ( m - 1) ⎞
1 + 2 n ⎜ ⎟ + 1 + 3n ⎜ ⎟ + 1 + ... + m n ⎜ ⎟ + 1⎪ ⎡1 1 1 1 1 1⎤
⎪ ⎝ 2⎠ ⎝ 3⎠ ⎝ m ⎠ ⎪ = lim ⎢ + + + ... + + + ⎥
⎨ lim 2 ⎬ n→∞ ⎣ n! ( n - 1)! ( n - 2)! ( 2)! 1! 1⎦
n→∞
⎪ m ⎪
⎪ ⎪ [∵ a1 = 1; given]
⎩ ⎭ ⎡ 1 1 1 ⎤
1 + 2 + 3 + ... + m = e ⎢as, e = 1 + + + + ... ∞ ⎥
= lim ⎣ 1! 2! 3! ⎦
m→∞ m2
The correct option is (B)
⎛ ⎛ 1⎞ n
⎛ 2⎞
n

⎜ ⎜ ⎟
∵ → 0 as n → ∞ ; ⎜⎝ ⎟⎠ → 0 as n → ∞; ... ; ⎡ ⎛ 1⎞ ⎛ 1 ⎞⎤
n
⎝ ⎝ 2⎠ 92. lim n - n ⎢( n + 1) ⎜ n + ⎟ ... ⎜ n + n -1 ⎟ ⎥
2
3
n→∞ ⎣ ⎝ 2 ⎠ ⎝ 2 ⎠⎦
⎛ m - 1⎞
n ⎞
⎟⎠ → 0 as n → ∞⎟
n
⎜⎝ ⎡ ⎛ 1⎞ ⎛ 1 ⎞⎤
⎢ ( n + 1) ⎜⎝ n + ⎟ ... ⎜ n + n -1 ⎟⎠ ⎥
m ⎠
2⎠ ⎝ 2
m ( m + 1) 1⎛ 1⎞ 1 = lim ⎢ ⎥
= lim = lim ⎜1 + ⎟ = n→∞ ⎢ nn ⎥
m →∞2m 2 m →∞ 2 ⎝ m⎠ 2 ⎢ ⎥
⎣ ⎦
The correct option is (D) n n
⎛ 1⎞ ⎛ 1 ⎞
89. We know, n+n n + n -1
⎛ n + 1⎞ ⎜ 2 ⎟ ⎜ 2 ⎟
1⎛ 1⎞ = lim ⎜ ⎟ ⋅⎜ ⎟ ... ⎜ ⎟
n→∞ ⎝ n ⎠
⎜1 - n ⎟⎠ ⎜
n
⎟ ⎜ n ⎟
n
1 2⎝ 2 ⎝ ⎠ ⎝ ⎠
∑ r = 1
i.e., sum of n terms of G.P.
r =12 1- n n n
⎛ 1⎞ ⎛ 1⎞ ⎛ 1 ⎞
2 = lim ⎜1 + ⎟ ⋅ ⎜1 + ⎟ ... ⎜1 + n -1 ⎟ (1∞ form)
n→∞ ⎝ n⎠ ⎝ 2n ⎠ ⎝ 2 n⎠
which tends to one as n → ∞ but always remains less than
one. 2n 2n -1. n
n
⎛ 1⎞ ⎛ 1⎞ 2 ⎛ 1 ⎞ 2n - 1
⎡ n 1⎤ = lim ⎜1 + ⎟ ⋅ ⎜1 + ⎟ ... ⎜1 + n -1 ⎟
n→∞ ⎝ n⎠ ⎝ 2n ⎠ ⎝ 2 n⎠
Thus, ⎢ ∑ r ⎥ → 0 as n → ∞
⎢⎣ r = 1 2 ⎥⎦
= e1 · e1/2 · e1/4 … e1/2n–1
⎪⎧ ⎫⎪
an
⎡ n 1⎤ ⎛ 1⎞
\ lim ⎢ ∑ r ⎥ = 0 … ⎨ using; lim ⎜1 + ⎟ = e a ⎬
n→∞ ⎝ n ⎠
n→∞ ⎢ ⎪⎩ ⎪⎭
⎣ r = 1 2 ⎥⎦ 1
The correct option is (A) 1-
1

90. Here, 0 < cos x < 1; if 0 – h < x < 0 + h = e(1 + 1/2 + 1/4 + …) = e 2 = e2
\ [cos x] = 0 The correct option is (B)
Hence, lim | x | [cos x ] x y - yx ⎛0 ⎞
x →0 93. lim ⎜⎝ form⎟⎠
x→ y xx - y y 0
= lim | x |0 = lim 1 = 1
x →0 x→0 yx y -1 - y x log y
= lim (applying L-Hospital’s rule)
The correct option is (B) x→ y x x (1 + x log x ) - 0
a yx y -1 - y x log y
91. We have, an – 1 + 1 = n (1) = lim
n x→ y x x (1 + x log x )
⎛ a + 1⎞ ⎛ a2 + 1⎞ ⎛ an + 1⎞
\ lim ⎜ 1 ⎟ ⎜⎝ a ⎟⎠ ... ⎜⎝ a ⎟⎠ y ⋅ y y -1 - y y log y
n→∞ ⎝ a ⎠
1 2 n
=
y y (log y + 1)
⎛a ⎞ ⎛ a3 ⎞ ⎛ a4 ⎞ ⎛ an +1 ⎞ 1 1 - log y
= lim ⎜ 2 ⎟ ⎜⎝ 3 ⎟⎠ ⎜⎝ ⎟⎠ ... ⎜⎝ ⎟ ⋅ =
n→∞ ⎝ 2 ⎠ 4 n + 1⎠ a1 ⋅ a2 ... an 1 + log y
an +1 1 + an The correct option is (A)
= lim = lim [using (1)]
n→∞ ( n + 1)! n→∞ n!
11.40 Chapter 11

n ⎛ 3 1⎞ cos 2 (1 - cos 2 (1 - cos 2 (1 ... cos 2 q ))


r -r+
94. lim ∑ cot ⎜ -1
r⎟
97. lim
q→ 0 ⎛ p ( q + 4 - 2⎞
n→∞
r =1 ⎜⎝ ⎟⎠ sin ⎜
2 q ⎟
n ⎝ ⎠
⎛ 2r ⎞
= lim
n→∞
∑ tan -1 ⎜⎝ 1 - r 2 + r 4 ⎟⎠ cos 2 (sin 2 (sin 2 ... (sin 2 q ))
r =1 = lim
q→ 0 ⎛ p ( q + 4 - 2⎞
-1 ⎛ ⎞
n
2r sin ⎜ ⎟
= lim ∑ tan ⎜ ⎟ ⎝ q ⎠
n→∞
r =1 ⎝ 1 - (r - r) (r + r) ⎠
2 2

n ⎛ (r 2 + r) - (r 2 - r) ⎞ cos 2 (sin 2 (sin 2 ... (sin 2 q )) cos 2 0


= lim = =
= lim
n→∞
∑ tan -1 ⎜ ⎟
⎝ 1 - (r 2 + r) (r 2 - r) ⎠
q→ 0 ⎛
q
⎞ sin
p
2
r =1
sin ⎜ p lim ⎟
( )
4
n ⎜⎝ q → 0 q q + 4 + 2 ⎟⎠
= lim
n→∞
∑ ⎡⎣tan -1( r 2 + r ) - tan -1( r 2 - r )⎤⎦
r =1 The correct option is (C)
–1 –1 –1 –1 –1
= lim [tan 2 – tan 0) + (tan 6 – tan 2) + (tan 12 –
n→∞ tan x - sin{tan -1(tan x )}
–1 –1 2 –1 2 98. lim f ( x ) = lim
tan 6) + … + {tan (n + n) – tan (n – n)}] x→
p
- x→
p
- tan x + cos 2 (tan x )
2 2
–1 2 –1
= lim {tan (n + n) – tan (0)} tan x - sin x
n→∞
p = lim
–1
= tan (∞) – tan (0) = –1 p
x→ - tan x + cos 2 (tan x )
2 2
[∵ tan–1(tan x) = x, where x < p/2]
n ⎛ 3 1⎞ p
r -r+
\ lim ∑ cot -1 ⎜ r⎟ = sin x
n→∞
r =1 ⎜⎝ ⎟⎠ 2 1-
2 tan x
= lim
The correct option is (C) x→
p cos 2 (tan x )
1+
95. We know n ≤ [x] < n + 1 ⇒ [x] = n 2
tan x
n sin x
Here, → n as x → 0 but less than n 1- 0
x = =1
n tan x 1+ 0
Also, → n as x → 0 but more than n
x ⎡ cos 2 (tan x ) finite quantity ⎤
⎢∵ lim = = 0⎥
⎡ n sin x ⎤ p tan x ∞
Thus, n – 1 ≤ ⎢ ⎥ < n as x → 0 ⎢ x→ - ⎥
⎣ x ⎦ ⎣ 2 ⎦
⎡ n sin x ⎤ sin x
⇒ ⎢ ⎥ =n–1 1+
⎣ x ⎦ Similarly, lim f ( x ) = lim tan x
p
x→ +
p
x→ + cos 2 (tan x )
⎡ n tan x ⎤ 2 2 1+
Again, n≤ ⎢ ⎥ < n + 1 as x → 0 tan x
⎣ x ⎦
[∵ tan–1 (tan x) = x – p, if x > p/2]
⎡ n tan x ⎤
⇒ ⎢ x ⎥ =n
⎣ ⎦ 1+ 0
= =1
⎛ ⎡ n sin x ⎤ ⎡ n tan x ⎤⎞ 1+ 0
Thus, lim ⎜ ⎢ + \ lim f ( x ) = 1
x →∞ ⎝ ⎣ x ⎥⎦ ⎢⎣ x ⎥⎦⎟⎠ x→
p
2
= (n – 1) + (n) = (2n – 1) The correct option is (A)
The correct option is (C) 99. Put c = a1/4 and z = x1/4, we get the function whose limit is
96. We know, required, as
x 8
→ 1 , as x → 0, but less than 1 ⎧⎡ 2 -1 ⎫
2 -1 ⎤
⎪ ⎢⎛ c + z ⎞
1
sin x 2cz log c4 ⎪
x ⎨ ⎜ ⎟ - 3 ⎥ - 2 2
4


Also, → 1 , as x → 0 but less than 1 ⎢
⎪ ⎣⎝ c - z ⎠ z - cz + c z - c
2 2 3⎥

tan x ⎩ ⎦ ⎭
x2 ⎧⎡ c2 + z 2 -1 ⎫
8
Thus, → 1 as x → 0, but less than 1. ⎪ 2cz ⎤ ⎪
sin x tan x = ⎨⎢ - ⎥ - c⎬
⎡ x2 ⎤ x2 ⎪⎩ ⎢⎣ c - z ( z - c) ( z 2 + c 2 ) ⎥⎦ ⎪⎭
Hence, lim ⎢ ⎥ = 0 as 0 ≤ < 1 as x → 0 8
= (c – z – c) = z = x8 2
x → 0 ⎢ sin x tan x ⎥ sin x tan x
⎣ ⎦
Hence, required limit as x → a = a2
The correct option is (A)
The correct option is (C)
Limits 11.41

(log (1 + x ) - log 2) (3.4 x -1 - 3 x ) 1 1


100. lim + x + O ( x2 )
x →1 {(7 + x )1/3 - (1 + 3 x )1/2} sin p x 2 3 1
= lim =
x→0 1+ x 2
[log ( 2 + t ) - log 2] [3.4t - 3(t + 1)
= lim [O (x2) means terms containing x2, x3, x4, …]
t →0 {(8 + t )1/3 - ( 4 + 3t )1/2} sin p (t + 1)
The correct option is (C)
[By putting x = 1 + t]
103. ∵ ax2 + bx+ c = 0 has roots a and b, therefore
⎛ t⎞ a b 1 1
log ⎜1 + ⎟ (3 ( 4t - 1) - 3t ) + + c = 0 i.e., cx2 + bx+ a = 0 has roots and
⎝ 2⎠ x 2
x a b
= - lim
t →0 ⎡
3t ⎞ ⎤ ⎛ 2 b a⎞ ⎛ 1⎞ ⎛ 1⎞
1/3 1/2
⎛ t⎞ ⎛ ⇒ c ⎜x + + ⎟ = c⎜x - ⎟ ⎜x - ⎟
⎢ 2 ⎜1 + ⎟ - 2 ⎜1 + ⎟ ⎥ sin p t ⎝
⎢⎣ ⎝ 8⎠ ⎝ 4 ⎠ ⎥⎦ cx c ⎠ ⎝ a⎠ ⎝ b⎠

⎛ t⎞ ⎡1 - cos (cx 2 + bx + a) ⎤
log ⎜1 + ⎟ Now, lim ⎢ ⎥
⎝ 2 ⎠ ⎡ 3 ( 4 - 1) ⎤
t
1 2 (1 - a x ) 2
= - lim ⋅ ⋅⎢ - 3⎥ x→
1 ⎢⎣ ⎥⎦
t →0 p t ⎢⎣ t ⎥⎦ a
1/ 2
2 ⎧ 2 ⎛ cx 2 + bx + a ⎞ ⎫
p

t
= lim ⎨
⎪ sin ⎜ ⎟⎪
⎝ 2 ⎠⎬
sin p t ⎡ t 3t ⎤ 1
x→ ⎪ ⎪
⎢1 + 24 - 1 - 8 + terms containing t , t , etc.⎥
2 3
a ⎩ (1 - a x ) 2

⎣ ⎦
3 9 4 ⎛ cx 2 + bx + a ⎞
= ·(3 log 4 - 3) = log sin ⎜ ⎟
p p e = lim ⎝ 2 ⎠
1
x→
The correct option is (A) a 1 - ax
(1 - x ) (1 - x 2 ) ... (1 - x 2 n ) ⎛c ⎛ 1⎞ ⎛ 1⎞⎞
101. lim sin ⎜ ⎜ x - ⎟ ⎜ x - ⎟ ⎟
x →1 [(1 - x ) (1 - x 2 ) ... (1 - x n )]2 ⎝ ⎠
⎝ 2 a ⎝ b ⎠⎠
= lim
(1 - x )(1 - x 2 )…(1 - x n )(1 - x n +1 )(1 - x n + 2 )…(1 - x 2 n ) x→
1 ⎛ 1⎞
-a ⎜ x - ⎟
lim a

x →1 [(1 - x )(1 - x 2 )…(1 - x n )]2 a⎠

(1 - x n +1 )(1 - x n + 2 )...(1 - x 2 n ) ⎛c ⎛ 1⎞ ⎛ 1⎞⎞ c⎛ 1⎞


= lim sin ⎜ ⎜ x - ⎟ ⎜ x - ⎟ ⎟ x- ⎟
x →1 [(1 - x )(1 - x 2 ) (1 - x 3 ) ... (1 - x n )] ⎝2 ⎝ a⎠ ⎝ b⎠⎠ 2 ⎜⎝ b⎠
= lim · lim
1 c⎛ 1⎞ ⎛ 1⎞ 1 -a
(1 - x n +1 )(1 - x n + 2 )...(1 - x 2 n ) x→
a ⎜⎝ x - ⎟⎠ ⎜ x - ⎟
x→
a
= lim 2 a ⎝ b⎠
x →1 (1 - x )(1 - x ) ... (1 - x )
(1 - x ) (1 - x ) ... (1 - x ) c ⎛ 1 1⎞
× 1· ⎜ - ⎟
(1 - x ) (1 - x ) ... (1 - x ) 2 n 2 ⎝ a b⎠ c ⎛ 1 1⎞
= = -
1 1 1 ( 2n)! -a 2a ⎜⎝ a b ⎟⎠
= (n + 1) (n + 2) … 2n · · ... =
1 2 n ( n!) 2 The correct option is (A)
The correct option is (C)
104. lim f ( x ) = lim {- h} cot {- h}
p x → 0- h→0
102. Since maximum value of cos–1x =
2
k
kp = lim (1 - h) cot (1 - h) = cot 1
∑ cos a r = 2 is possible if and only if each
-1 h→0
tan 2 {h}
r =1
p lim f ( x ) = lim
cos–1 a r = ⇒ ar = 0 x→0 +
h→0 h2 - [h]2
2
tan 2 h2
k = lim =1
∑ (a r )
r
\ q= =0 h→0h2
r =1
\ lim f ( x ) does not exist,
x→0
(1 + x 2 )1/3 - (1 - 2 x )1/4
\ lim The correct option is (D)
x→q x+x 2

x a sin b x a + b - c ⎛ sin x ⎞
b
⎛ xc ⎞
(1 + x )2 1/3
- (1 - 2 x )1/4 105. lim = lim x
= lim x→0 sin ( x c ) x→0 ⎝⎜ x ⎠⎟ ⎜⎝ sin ( x c ) ⎟⎠
x→ 0 x + x2
The above limit is non-zero if a + b – c = 0
⎛ 1 2 4 ⎞ ⎛ x 2 ⎞
⎜⎝1 + x + O ( x )⎟⎠ - ⎜⎝1 - + O ( x )⎟⎠ The correct option is (D)
3 2
= lim
x→ 0 x (1 + x )
11.42 Chapter 11

More than One Option Correct Type


106. We know that |cosq | ≤ 1 for all q. ⎛ a y + b y + c y - 3⎞
ln ⎜ 1 + ⎟
So, if |cos n! px| < 1, ay + by + cy - 3 ⎝ 3 ⎠
= lim ⋅
lim lim (1 + cos 2 m n!p x ) = (1 + 0) = 1 y→0 y a +b +c -3
y y y
m →∞ n→∞
and if |cosn! px| = 1, 3
⎛ a - 1 b - 1 c - 1⎞
y y y
ln (1 + t )
lim lim (1 + cos 2 m n!p x ) = lim lim (1 + 12 m ) = lim ⎜ + + ⋅ lim
m →∞ n→∞ m →∞ n→∞ y→0 ⎝ y y y ⎟⎠ t → 0 t
= lim lim (1 + 1) = 2 ⎡ ay + by + cy - 3 ⎤
m →∞ n→∞
⎢ Putting = t⎥
The correct option is (A) and (B) ⎢⎣ 3 ⎥⎦
(1 + a3 ) + 8e1 x ⎛ ∞ ⎞ = ln a + ln b + ln c = ln(abc)
107. We have 2 = lim ⎜ form⎟⎠ (1) Hence, the required limit is eln (abc) = abc
x → 0 1 + (1 - b3 ) e1 x ⎝ ∞
The correct option is (C) and (D)
0 + 8e1 x ( -1 x 2 )
⇒ 2 = lim 109. Using the expansion, we have,
x → 0 0 + (1 - b3 ) e1 x ( -1 x2 )
⎛ x2 ⎞ ⎛ x 2 x3 ⎞
[Using L’Hospital’s rule] ax ⎜1 + x + + …⎟ - b ⎜ x - + - …⎟
⎝ 2 ! ⎠ ⎝ 2 3 ⎠
⇒ 1 – b3 = 4 ⇒ b3 = –3 ⇒ b = (–3)1/3
⎛ x x 2 x3 ⎞
\ From (1), + cx ⎜1 - + - + …,⎟
⎝ 1! 2! 3! ⎠
(1 + a3 ) + 8e1 x lim
2 = lim x→0 ⎛ x 3
x ⎞
5
x→0 1 + 4e1 x x2 ⎜ x - + …
3 ⎝ 3! 5! ⎟⎠
⇒ 1 + a = 2 i.e., a = 1
Hence a = 1 and b = (–3)1/3 ⎛ b ⎞ ⎛ a b c⎞
x ( a - b + c ) - x 2 ⎜ a + - c⎟ + x 3 ⎜ - + ⎟ + …
The correct option is (B) and (C) ⎝ 2 ⎠ ⎝ 2 3 2⎠
⇒ lim
3x x→0 ⎛ x 3
x 5 ⎞
⎛ a1/x + b1/x + c1/x ⎞ x2 ⎜ x - + - …⎟
108. lim ⎜ ⎟ ⎝ 3! 5! ⎠
x→∞ ⎝ 3 ⎠
3/ y
=2
⎛ ay + by + cy ⎞ ⎡ 1 ⎤ Now, above limit would exist if least power in numerator is
= lim ⎜ ⎟ ⎢ Putting x = y ⎥
y→0 ⎝ 3 ⎠ ⎣ ⎦ greater than or equal to least power in denominator
i.e., coefficient of x and x2 must be zero and coefficient of x3
3 ⎛ ay +by +cy ⎞ should be 2.
ln ⎜
⎝ 3 ⎠⎟
= lim e y b a b c
y→0 i.e., a – b + c = 0, a + – c = 0, - + = 2
2 2 3 2
Now, we have, On solving, we get a = 3, b = 12, c = 9
3 ⎛ ay + by + cy ⎞ The correct option is (A), (B) and (C)
lim ln ⎜ ⎟
y→0 y ⎝ 3 ⎠
3 ⎛ a y + b y + c y - 3⎞
= lim ln ⎜1 + ⎟
y→0 y ⎝ 3 ⎠

Passage Based Questions


2 cos 2 x - 2 cos x ⎛ 0⎞
sin 2 x + a sin x ⎛ 0⎞ Hence, k = lim 2 ⎜⎝ ⎟⎠
110. Let k = lim x→0 3x 0
⎜⎝ ⎟⎠
x→0 x3 0
-4 sin 2 x + 2 sin x ⎛ 0⎞
2 cos 2 x + a cos x = lim ⎜⎝ ⎟⎠
= lim x→0 6x 0
x→0 3x 2
-8 cos 2 x + 2 cos x -8 + 2
[Using L’Hospital’s rule] = lim = = –1
x→0 6 6
We require 2 cos2x + acosx = 0 for x = 0 as denominator is
zero. The correct option is (B)
\ a = –2
Limits 11.43

p a-x Hence, by Sandwich Theorem, we have


111. lim a 2 - x 2 cot (0 ⋅ ∞ form)
x→a 2 a+ x 1•3•5 ... ( 2n - 1)
lim =0
n → +∞ 2 •4 •6 ... 2n
a2 - x 2 ⎛0 ⎞ The correct option is (C)
= lim ⎜⎝ form⎟⎠
x→a p a-x 0 115. Since, 0 ≤ {rx} < 1 for r = 1, 2, 3, …, n
tan
2 a+ x n n n
⇒ 0≤ ∑ {rx} < ∑ (1) ⇒ 0 ≤ ∑ {rx} <n
-2 x r =1 r =1 r =1

2 a2 - x 2 Dividing throughout by n2, we have


= lim
x→a p a-x p 2a n
- sec 2 × ×
2 a + x 2 2( a + x ) a 2 - x 2 0
∑ {rx} 1
r =1
≤ <
n2 n2 n
4a n
=
p ∑ {rx} 1
r =1
The correct option is (C) ⇒ lim 0 ≤ lim 2
< lim
n→∞ n→∞ n n→∞ n
log e | x | n
lim
lim | x | sin x = lim esin x log | x | = e ∑ {rx}
x→0
cosec x
112. e

x→0 x→0 r =1
⇒ 0 ≤ lim <0
lim
1/ x
lim
sin 2 x n→∞ n2
= e
x→0
= e
x→0
cos ec x cot x x cos x
{x} + {2 x} + ... + {nx}
\ 0 ≤ lim <0
2
⎛ sin x ⎞ ⎛ x ⎞
n→∞ n2
lim
x→0 ⎜

⎝ x ⎟⎠ ⎜⎝ cos x ⎟⎠
= e = e–(1)2 · (0) = e0 = 1 According to Sandwich Theorem or Squeeze Principle
The correct option is (B) {x} + {2 x} + ... + {nx}
lim =0
1 ( x -a )
n→∞ n2
113. lim (1 + ax + bx + c) 2
x →a The correct option is (B)
1 116. We have,
lim ⎡(1+ ax 2 + bx + c ) -1⎤⎦
x →a
( x -a ) ⎣ 12 · xx– 1 ≤ [12xx] < 12xx x – 1 ≤ [x] < x]
= e [∵
( ax + bx + c )
2
a ( x - a )( x - b )
22 · xx– 1 ≤ [22xx] < 22xx
lim lim
x →a
( x -a ) x →a
( x -a ) …
= e = e

[∵ a, b are roots of ax2 + bx + c = 0]
a ( a – b) n2 · xx– 1 ≤ [n2xx] < n2 · xx
=e .
Adding the above inequations,
The correct option is (B)
x x Sn 2 - n S [ n2 x x ] x x Sn 2
114. We can see that
3
≤ 3

n n n3
1· 3· 5 ... ( 2n - 1) 1 1 1 1
> · · … to n terms = n n ( n + 1) ( 2n + 1)
2 · 4 · 6 ... 2n 2 2 2 2 1 S [ n2 x x ]
⇒ xx 6 n3 – 2

n n3
1· 3· 5 ... ( 2n - 1) ⎛ 1⎞ ⎛ 1⎞ ⎛ 1⎞
and, = ⎜1 - ⎟ ⎜1 - ⎟ ... ⎜1 - ⎟ n ( n + 1) ( 2n + 1)
2 · 4 · 6 ... 2n ⎝ 2⎠ ⎝ 4⎠ ⎝ 2n ⎠ ≤ xx
6 n3
⎛ 1⎞⎛ 1⎞ xx S [ n2 x x ] xx
< ⎜1 - ⎟ ⎜1 - ⎟ … to n terms Now, applying lim , we have ≤ ≤
⎝ 2n ⎠ ⎝ 2n ⎠ n→∞ 3 n3 3
n Hence, by Sandwich Theorem, we have
⎛ 1⎞
= ⎜1 - ⎟ S [ n2 x x ]
xx
⎝ 2n ⎠ lim =
n→∞ n3 3
Thus, we have,
n Now, the required unit
1 1· 3· 5 ... ( 2n - 1) ⎛ 1⎞
< < ⎜1 - ⎟ ⎛
2 n
2 · 4 · 6 ... 2 n ⎝ 2 n⎠ S [ n2 x x ] ⎞ 1 1
lim ⎜ lim
n→0 ⎝ n→∞ 3 ⎟ = 3 xlim xx =
⎠ → +
+
Now, we have, n 0 3
n
1 ⎛ 2n - 1⎞ The correct option is (B)
lim n = 0 and lim ⎜ ⎟ =0
n → +∞ 2 n → +∞ ⎝ 2n ⎠
11.44 Chapter 11

Match the Column Type


⎡⎛ 1⎞
13 ⎤ - hh - 2hh ln h - hh - 1 - hh (ln h) 2
(I) lim ⎡ n2 - n3 + n⎤ = lim n ⎢⎜ -1 + ⎟ + 11 3 ⎥
3
117. = lim

n→∞ ⎣ ⎥⎦ n→∞ ⎢ ⎝ n ⎠ ⎥⎦ h →1 -1 / h2

⎛1 ⎞ -1 - 1
⎜⎝ - 1⎟⎠ + 1 = =2
n -1
= lim n ⋅ 23 13
n→∞ ⎛1 ⎞ ⎛1 ⎞ The correct option is (A)
⎜⎝ - 1⎟ + 1 - ⎜⎝ - 1⎟ (II) Required limit =
n ⎠ n ⎠
n
5r + 2 r n ⎧⎪⎛ 1 ⎞ r ⎛ 1 ⎞ r ⎫⎪

⎢ Using a + b = 2
a3 + b3 ⎤

lim
n→∞
∑ 10 r
= lim
n→∞
∑ ⎨⎜⎝ 2 ⎟⎠ + ⎜⎝ 5 ⎟⎠ ⎬
a - ab + b 2 ⎥⎦
r =1 ⎪
r = 1⎩ ⎪⎭
⎢⎣
1 1 1 ⎧ ⎛ 1⎞
n
⎛ 1⎞ ⎫
n

= lim = = ⎪⎪ 1 1 - ⎜⎝ ⎟⎠ 1- ⎜ ⎟ ⎪
⎝ 5⎠ ⎪
n→∞ ⎛ 1 ⎞
23
⎛1 ⎞
13
1+1+1 3 = lim ⎨ ⋅
2
+
1

⎜⎝ - 1⎟⎠ + 1 - ⎜⎝ - 1⎟⎠ n → ∞⎪2 1 5 1 ⎪
n n 1- 1-
⎪⎩ 2 5 ⎪ ⎭
The correct option is (C) 1 5
3
= 1+ =
x2 - 2 3 x + 1 4 4
(II) lim The correct option is (D)
x →1 ( x - 1) 2
⎡ x +1 p ⎤
y2 - 2 y + 1 (III) lim x ⎢ tan -1 -
= lim x = y; as x→ 1, y→ 1] x + 2 4 ⎥⎦
3
[Putting x →∞ ⎣
y →1 ( y 3 - 1) 2
⎡ x +1 ⎤
( y - 1) 2 1 1 = lim x ⎢ tan -1 - tan -1 1⎥
= lim = lim = x →∞ ⎣ x+2 ⎦
- 1) 2 ( y 2 + y + 1) 2
y →1 ( y y →1 ( y 2 + y + 1) 2 9
The correct option is (A) ⎛ x +1 ⎞
-1
-1 ⎜ ⎟
(III) (n – 2)th factor of the series is = lim x tan ⎜ x + 2 ⎟
x →∞ x +1
n - 1 n2 + n + 1 ⎜1+ ⎟
tn = ⋅ ⎝ x + 2⎠
n + 1 n2 - n + 1
⎛ -1 ⎞
= lim x tan -1 ⎜
Therefore, required limit = lim t3t 4t5 ... t n - 2t n - 1t n
n→∞
x →∞ ⎝ 2 x + 3 ⎟⎠
⎡⎛ 2 3 4 n - 3 n - 2 n - 1⎞ ⎛ 1 ⎞
tan -1 ⎜
= lim ⎢⎜ ⋅ ⋅ ...
n→∞ ⎝ 4 5 6 n -1

n

n + 1⎟⎠ ⎝ 2 x + 3 ⎟⎠ 1
⎣ = – lim ⋅
x →∞ ⎛ 1 ⎞ ⎛ 3⎞
⎛ 13 ⎞ 21 31 n + n + 1⎤ 2+ ⎟
⎝⎜ 2 x + 3 ⎠⎟ ⎝⎜
2
⋅⎜ ⎟ ⋅ ⋅ … 2 ⎥ x⎠
⎝ 7 ⎠ 13 21 n - n + 1⎥⎦
1 1
= –1 × =–
2 ⋅ 3 n2 + n + 1 6 2 2
= lim ⋅ =
n → ∞ n( n + 1) 7 7 The correct option is (B)
The correct option is (B) nk sin 2 ( n!) nk sin 2 ( n!)
n ⎛ x ⎞ (IV) lim = lim
⎛ x⎞ lim n ⎜ cos -1⎟
⎝ n ⎠
n→∞ n+2 n→∞ ⎛ 2⎞
(IV) lim ⎜ cos ⎟ = e
n→∞
n ⎜1 + ⎟
n→∞ ⎝ n⎠ 2
⎝ n⎠
⎛ ⎛ x ⎞⎞
⎜ sin ⎜⎝ 2 n ⎟⎠ ⎟ x 2
-2 lim ⎜ ⎟ ⋅ 2 ⋅n sin 2 ( n!)
⎛ x⎞ n →∞
⎜ x ⎟ 4n = lim
- n⋅2 sin ⎜ ⎟ n → ∞ 1- k ⎛ 2⎞
2
lim ⎜⎝ 2 n ⎟⎠
⎝ 2n⎠ n ⎜1 + ⎟
= e = e
n →∞

x2 ⎝ n⎠
-2 × lim
= e = e0 = 1
n →∞
4n
a finite quantity
The correct option is (D) =

118. (I) Let sin x = h, then as x → p/2, h → 1 [∵ sin2 (n !) always lies between 0 and 1. Also,
\ given limit since 1 – k > 0,
h - hh 1 - hh - hh ln h \ n1 – k → ∞ as n → ∞]
= lim = lim
h → 1 1 - h + ln h h →1 -1 + 1 / h =0
[Using L’ Hospital rule] The correct option is (C)
Limits 11.45

Assertion-Reasoning Type
12 + 22 + 32 + ... + r 2 e y - 1 ln (1 + x ) - x
119. tr = = lim - e ⋅ ⋅
13 + 23 + 33 + ... + r 3 x→0 y x2
2
r ( r + 1)( 2r + 1) ⎛ 2 ⎞ 2⎛1 1 ⎞ ⎡ ln (1 + x ) - x ⎤
= ⋅⎜ = ⎜ + ⎟ ⎢ Putting = y⎥
6 ⎝ r ( r + 1) ⎟⎠ 3 ⎝ r r + 1⎠ ⎣ x ⎦
Now, we have,
2 ⎡ ⎛ 1 ⎞ ⎛ 1 1⎞ ⎛ 1 1 ⎞ ⎛1 1 ⎞⎤
\ Sn = ⎢ - ⎜1+ ⎟ + ⎜ + ⎟ - ⎜ + ⎟ + … ± ⎜ + ln (1 + x ) - x ⎛ 0 ⎞
3 ⎣ ⎝ 2 ⎠ ⎝ 2 3⎠ ⎝ 3 4 ⎠ ⎝ n n + 1⎟⎠ ⎥⎦ lim ⎜⎝ ⎟⎠
x→0 x 0
2⎛ 1 ⎞ 1
= ⎜ -1 ± ⎟ -1
3⎝ n + 1⎠ 1+ x
= lim =0
2 x→0 1
\ lim Sn = – ln (1 + x ) - x ⎛ 0 ⎞
n→∞ 3 Also, lim ⎜⎝ ⎟⎠
The correct option is (D) x→0 x2 0
1
120. We have, -1
1+ x -1 1
x1 = 3, xn + 1 = 2 + xn = lim = lim =–
x→0 2x x → 0 2 (1 + x ) 2
x2 = 2 + x1 = 2+3 = 5
Hence, the required limit is
x3 = 2 + x2 = 2+ 5 ey -1 ln (1 + x ) - x
= –e lim ⋅ lim
\ x1 > x2 > x3 y→0 y x→0 x2
It can be easily shown by mathematical induction that the 1 e
= –e · 1 · – =
sequence x1, x2, …xn, … is a monotonically decreasing 2 2
sequence bounded below by 2. So, it is convergent. The correct option is (A)
Let lim xn = x. Then, 1
n→∞
123. (
lim [ f ( x )] + x 2
x→0
) { f ( x )}

xn + 1 = 2 + xn [ f ( x )] + x 2 - 1
⎡⎛ 1 ⎞⎤ { f ( x )}
⇒ lim xn + 1 =
n→∞
2 + lim xn
n→∞
x → 0 ⎢⎝
( )
= lim ⎢⎜ 1 + [ f ( x )] + x 2 - 1 [ f ( x )] + x
2
-1 ⎥

⎠ ⎥⎦
⇒ x= 2+ x ⎣
Now, we have,
⇒ x2 – x – 2 = 0
⇒ (x – 2)(x + 1) = 0 ⎡ tan x ⎤
⎥ + x -1 = 1 + 0 – 1 = 0
2
lim ⎢
⇒ x=2 (∵ xn > 0 ∀ n, \ x > 0) x→0 ⎣ x ⎦
The correct option is (A) [ f ( x )] + x 2 - 1
and, lim
n -1 ⎞ x→0 { f ( x )}
1⎛
⎜1 + e + e + ... + e
1/ n 2/ n
121. lim n

n→∞ n
⎝ ⎠ ⎡ x2 2x4 ⎤
⎢1 + + + ...⎥ + x 2 - 1
( )
n
1 1- e
1/ n 3 15
= lim ⋅ = lim
1- e = lim ⎣ ⎦
n→∞ n 1 - e1/ n n→∞ ⎡ 1 1 1 ⎤ x→0 ⎧⎪ x 2
2x4 ⎫⎪
n ⎢1 - 1 - - ⋅ 2 ...⎥ ⎨1 + + + ...⎬
⎣ n 2! n ⎦ ⎪⎩ 3 15 ⎪⎭
1- e 1- e ⎡ ⎤
= lim = =e–1 x3 2 x5
n→∞ 1 1 -1 ⎢ tan x = x + + + ...⎥
-1 - ⋅ ... ⎢⎣ 3 15 ⎥⎦
2! n
The correct option is (A) 1 + x2 - 1 1
= lim 2 5
= lim
ln (1 + x ) x→0 x 2x x→0 1 2x
+ + ... + + ...
e - (1 + x )1/x e-e x
3 15 3 15
122. lim = lim =3
x→0 x x→0 x
ln (1 + x ) - x \ Required limit = e3
e x -1 The correct option is (D)
= lim - e ⋅
x→0 x
11.46 Chapter 11

1 x 1 x 1 x
tan + 2 tan n + ... + n tan n cot q tan -1( m tan q ) - m
124. lim
n→∞
= lim +m
2 2 2 2 2 2 q →0 sin 2 (q/2)
⎛ 1 x⎞ 1 x tan -1( m tan q ) - m tan q
= lim - cot x + ⎜ cot x + tan ⎟ + 2 tan 2 + = lim +m
n→∞ ⎝ 2 2⎠ 2 2 q →0 tan q sin 2 (q/2)
1 x
... + n tan n tan -1( m tan q ) - m tan q
2 2 = lim +m
⎛1 x 1 x⎞ 1 x q →0 q3 / 4
= lim - cot x + ⎜ cot + 2 tan 2 ⎟ + ... + n tan n x - tan x
n→∞ ⎝2 2 2 2 ⎠ 2 2 = lim +m
x→0 x3
⎛ 1 x 1 x⎞ 1 x 3
= lim - cot x + ⎜ 2 cot 2 + 3 tan 3 ⎟ + ... + n tan n 4m
n→∞ ⎝2 2 2 2 ⎠ 2 2 [Putting m tan q = tan x; as q→ 0, x → 0]
⎡ 1 q 1 q⎤ x - tan x
⎢ repeatedly using cot q + 2 tan 2 = 2 cot 2 ⎥ = m + 4m3 xlim
⎣ ⎦ →0 x3
1 x Now, we have,
= lim - cot x + n cot n
n→∞ 2 2 x - tan x ⎛ 0 ⎞ 1 - sec 2 x ⎛ 0 ⎞
lim ⎜ ⎟ = lim ⎜⎝ ⎟⎠
1 ⎛ x/2 ⎞ n
x→0 x 3 ⎝ 0 ⎠ x → 0 3x 2 0
= lim - cot x +
n→∞ x ⎜⎝ tan x/2n ⎟⎠ -2 sec 2 x tan x -1
= lim =
1 q 1 x→0 6x 3
= –cot x + lim = –cot x +
x q → 0 tan q x Hence, the required limit is m – (4/3) m3
The correct option is (A) The correct option is (C)
cot q tan -1( m tan q ) - m cos 2 (q/2)
125. lim
q →0 sin 2 (q/2)
cot q tan -1( m tan q ) - m [1 - sin 2 (q/2)]
= lim
q →0 sin 2 (q/2)

Previous Year’s Questions

126. Key Idea: Limit of a function exists only, if


127. Key Idea : lim (1 + λ x )1/ x = e λ
LHL = RHL. x →∞
x
1 - cos 2 x 1 - 1 + 2 sin x 2 ⎛ x 2 + 5 x + 3⎞
Now, lim = lim Now, the limit lim ⎜ 2 ⎟
x→0 2x x→0 2x x →∞ ⎝ x + x + 2 ⎠

2 sin x sin x 4x + 1 ⎞
x
= lim = lim ⎛
x→0 x→0 x = lim ⎜1 + 2 ⎟
2x x →∞ ⎝ x + x + 2⎠
sin x ( 4 x +1) x
Let f ( x ) = ( 4 x +1) ⎤ x 2 + x +2
x ⎡ 1
⎢ ⎛ 4x + 1 ⎞ x + x+2 ⎥
2

sin( 0 - h) = lim ⎜1 + 2 ⎟
LHL = lim x →∞ ⎢⎝ x + x + 2⎠ ⎥
0-h
h→ 0 ⎢⎣ ⎥⎦
Now,
sin h ⎛ 1⎞
4+
= lim = -1 ⎝⎜ x ⎠⎟
h→ 0 - h lim
x →∞ 1 2
1+ - 2
sin( 0 + h) =e x x
RHL = lim
h→ 0 0+h = e4
and
sin h The correct option is (A)
= lim =1
h→ 0 h x
⎛ x - 3⎞
∵ LHL ≠ RHL 128. The limit lim ⎜ ⎟
x →∞ ⎝ x + 2 ⎠
sin x
\ lim does not exist. ⎡ 5 ⎤
x
x→0 x = lim ⎢1 -
The correct option is (D) x →∞ ⎣ x + 2 ⎥⎦
Limits 11.47

⎛ -5 x ⎞
131. By applying L’Hopital Rule, the given limit equals
⎡ ⎛ 5 ⎞ ⎤ ⎜⎝ x + 2 ⎠⎟
1/ ⎜ ⎟ 1 1
⎢⎛ ⎛ -5 ⎞ ⎞ ⎝ x + 2 ⎠ ⎥ +
= lim ⎢⎜1 + ⎜ ⎟ ⎟ ⎥ 3 + x 3 - x =2.
x →∞ ⎝
⎢ ⎝ x + 2 ⎠⎠ ⎥
lim
x→0
⎣ ⎦ 1 3

⎛ 5 ⎞ The correct option is (C)


lim ⎜ - 1+ 2 / x ⎠⎟
x →∞ ⎝
=e 132. Applying L. Hospital’s Rule
= e -5 f ( a) g ′( a) - g ( a) f ′( a)
lim =4
Alternative Method: x→ 2a g ′( a) - f ′ ( a)
x
⎛ x - 3⎞ k ( g ′( a) - ff ′( a))
lim ⎜ ⎟ =4
x →∞ ⎝ x + 2 ⎠ ( g ′( a) - f ′( a))
x k = 4.
⎛ 3⎞
⎜⎝1 - x ⎟⎠ The correct option is (A)
= lim x
x →∞ ⎛ 2⎞ 2x
⎛ a b⎞
⎜⎝1 + x ⎟⎠ 133. The limit lim ⎜1 + + 2 ⎟ =
x →∞ ⎝ x x ⎠
e -3 ⎛ 1 ⎞
= 2
= e -5 ⎜ a b ⎟ × 2 x × ⎛⎜ + 2 ⎞⎟
a b
e ⎛ a b ⎞ ⎜⎝ x + x 2 ⎟⎠ ⎝x x ⎠
lim ⎜1 + + 2 ⎟ = e2a
The correct option is (C) x →∞ ⎝ x x ⎠
xf ( 2) - 2 f ( x ) ⇒ a = 1, b ∈ R
129. The limit lim
x→2 x-2 The correct option is (B)
xf ( 2) - 2 f ( x ) + 2 f ( 2) - 2 f ( x )
= lim 134. Let L =
x→2 x-2
⎛ ( x - a )( x - b ) ⎞
f ( 2)( x - 2) - 2{ f ( x ) - f ( 2)} 2 sin 2 ⎜ a ⎟⎠
= lim 1 - cos a( x - a )( x - b ) ⎝ 2
x→2 x-2 lim = lim
x →a ( x - a )2 x →a ( x - a )2
f ( x ) - f ( 2)}
= f ( 2) - 2 lim
x→2 x-2 ⎛ ( x - a )( x - b ) ⎞
sin 2 ⎜ a ⎟⎠
= f ( 2) - 2 f ′( 2) = 4 - 2 × 4 = -4 2 ⎝ 2 a2 ( x - a )2 ( x - b )2
= lim × ×
x →a ( x - a )2 a2 ( x - a )2 ( x - b )2 4
Alternative Solution:
4
⎧ xf ( 2) - 2 f ( x ) ⎫
lim ⎨ ⎬ a 2 (a - b ) 2
x→2 ⎩ x-2 ⎭ Then, the limit L = .
2
= lim{ f ( 2) - 2 f ′( x )} ( by L′ Hopital’s Rule)
x→2 The correct option is (A)
= f (2) − 2f ′(2) 135. f (x) is a positive increasing function
=4−2×4=−4 ⇒ 0 < f (x) < f (2x) < f (3x)
The correct option is (C)
f ( 2 x ) f (3 x )
⇒ 0 <1< <
130. The limit, by applying L’Hopital rule, f ( x) f ( x)
⎛ p x⎞ f (2 x) f (3 x )
tan ⎜ - ⎟ (1 - sin x ) ⇒ lim 1 ≤ lim ≤ lim
⎝ 4 2⎠ x →∞ x →∞ f ( x ) x →∞ f ( x )
lim
x →p / 2 ⎛ p x⎞
4 ⎜ - ⎟ (p - 2 x ) 2 By sandwich theorem.
⎝ 4 2⎠
f (2 x)
( - cos x ) ⇒ lim =1
= lim x →∞ f ( x )
x →p / 2 8( -2)(p - 2 x )
The correct option is (D)
- sin x
= lim
x →p / 2 16( -2) 2 sin 2 ( x - 2)
136. lim
1 x→2 x-2
= .
32 2 | sin( x - 2) |
The correct option is (C) lim
x→2 x-2
11.48 Chapter 11

1
R.H.L. = 2 , L.H.L. = - 2 ⇒ a = , b = -2
2
Limit does not exist. x4
The correct option is (D) ⇒ f ( x) = - 2 x3 + 2 x 2
2
⇒ f ( x ) = 8 - 16 + 8 = 0.
(1 - cos 2 x )
137. lim
x→0
(3 + cos x ) The correct option is (A)
x(tan 4 x )
2 139. The value of the limit
⎛ sin x ⎞ 1 ⎛ 4 x ⎞
= lim ⎜
x →0 ⎝ x ⎠
⎟ ⋅ 4 ⎜ tan 4 x ⎟ ( 3 + cos x ) 2 sin 2 x × (3 + cos x ) 2 × 4
⎝ ⎠ lim = =2.
1 x→0 ⎛ tan 4 x ⎞ 4
= 2 × 1 × × 1 × ( 3 + 1) = 2. x×⎜
⎝ rx ⎟⎠
× 4 x
4
The correct option is (C) The correct option is (B)

⎛ x 2 + f ( x) ⎞
2
1 ⎛ tan x ⎞
⎟ =3.
138. Given that lim ⎜ lim ⎜ ⎟
2⎝
x → 0+ x ⎠
x→0 ⎝ x2 ⎠ 140. We have p = e = e

Since limit exits, the expression x 2 + f ( x ) = ax 4 + bx 3 + 3 x 2 1


\ log p =
⇒ f ( x ) = ax 4 + bx 3 + 2 x 2 2
The correct option is (D)
⇒ f ′( x ) = 4 ax 3 + 3bx 2 + 4 x

Also, f ′( x ) = 0 at x = 1, 2

You might also like